Water Resource Engineering

Download as pdf or txt
Download as pdf or txt
You are on page 1of 732

Water Resources Engineering –II

ODD Semester 2021 (5th Semester)


Module Course Modules and Lesson Plan Contact hours
No.

1. Irrigation: Necessity, Types, Techniques, Quality of Irrigation 3


water, Soil-water-plant relationship.
2. Water requirement of crops: Base period, Duty, Delta, 5
Commanded area, Intensity of irrigation, Consumptive use of
water, ETO, estimation of ETO, Modified Penman’s equations,
CropWAT software – examples, Irrigation efficiencies, Frequency
of irrigation.
3. Canal Irrigation: Classification of canals, types, layout of 7
irrigation canals, lined canals and unlined canals, design of
unlined canals: Kennedy's method, Lacey's method, design of
lined canals, typical sections, advantages of lining, materials
used, economics of canal lining.Design and maintenance of open
drains, closed drains, discharge and spacing of closed drains.
4. Storage for Irrigation: Flow mass curve, estimation of storage 2
required for irrigation and other demands.
5. Diversion Headworks: Necessity and uses, different types, 5
Layout and different components, weirs on permeable
foundation, Creep theories, Khosla's method.
6. Dam:Introduction: Definition, types, and necessity of 5
hydraulic structures; Dams, Barrages
Site Selection: Investigation survey, Selection of dam site,
Selection of type of dam, Classification, Geological
Investigation, Gravity Dam, Earthen Dam
8. Floods: Introduction, definitions, flood scenario in India, 3
concept of flood hazard mitigation – structural and
non-structural measures, flood plain zoning and
management
9. Flood Frequency Analysis: Frequency studies using 4
different distributions, Risk analysis, Design flood.
10. Flood Routing: Passage of flood through reservoir and 5
streams, hydraulics of flood wave propagation, hydrologic
storage routing, channel routing.
Total 39
Irrigation
What?
artificial application of water
Where to apply?
to the root zone of the crop
When to apply?
at right time
How much to apply?
in required quantity
What for?
for optimum crop yield
IRRIGATION ENGINEERING
• IT IS A SCIENCE DEALING WITH PLANNING , DESIGNING,
CONSTRUCTION, OPERATION AND MAINTENANCE OF IRRIGATION
WORKS.
Why Irrigation is Necessary
❖ Inadequate Rainfall
❖ Uneven Distribution of Rainfall
❖ Development of Farming in Desert area
❖ Growing a number of crops in a year
❖ Growing Superior Crops
Benefit of Irrigation
• Increase in yield of crops
• Protection from famine
• Elimination of mixed cropping
• Improvement of cash crops
• General prosperity of the farmers and country
• Generation of Hydro-electric power
• Inland navigation
• Means of communication
• Water Supply
• Fishery
• Afforestation
ILL Effects of Irrigation
• Rising Water Table: Water-logging
• Problem of water pollution
• Formation of marshy land: Mosquitoes
• Dampness in weather
• Loss of valuable land
Systems of Irrigation
The classification of Irrigation systems can be done
based on the way water is supplied to the land
Irrigation systems can be broadly classified into
Surface Irrigation and
Sub-surface Irrigation

Sub-surface Irrigation can be divided into two types:


Natural Sub-irrigation
Artificial Sub-irrigation
Components of a surface irrigation system
Surface Irrigation Systems
Lift Irrigation:
When water is lifted from surface sources or underground
sources by man or animal power, mechanical or electrical
power and directly supplied to the agricultural land, it is
known as lift irrigation.
In this method, isolated small areas can be irrigated.
The vast areas can not be included in this system.
Shallow well draws water from the unconfined aquifer.
The yield of shallow well is dependant on the water
table . In dry season when water table goes below the
bed of the well, the well is dried up.

Deep well draws water from confined aquifer. The


discharge is reliable throughout the year.
Flow Irrigation:
When water flows under gravitational pull through the
canal towards the agricultural land. It is termed as
flow irrigation. In this system, the head of the canal
should always be at higher elevation than the land to
be irrigated.
• Direct Irrigation System
• Storage Irrigation System
Components of a surface irrigation system
Methods of Distribution of Water

Surface Subsurface
methods methods

Sprinkler Drip
irrigation irrigation
a) Surface Irrigation: Just flooding water. About 90% of
the irrigated areas in the world are by this method.

b) Sprinkler Irrigation: Applying water under pressure.


About 5 % of the irrigated areas are by this method.

c) Drip or Trickle Irrigation: Applying water slowly to the


soil ideally at the same rate with crop consumption.

d) Sub-Surface Irrigation: Flooding water underground


and allowing it to come up by capillarity to crop roots.
Drip
Irrigation
1. Water Requirement of Crops
2. Design of Source of Water
3. Design of Canal
4. Design of Distribution System
5. Design of Drainage
6. Design of Structures
IRRIGATION PROJECT SURVEYING
When a new irrigation project is to be taken up, the
following points may be considered:

(1) Preliminary Survey or Reconnaissance Survey:


It is preliminary site visit of the proposed area to note
down various preliminary points and quick check up
whether the irrigation project is feasible or not ?
(2) Availability of Irrigation Water:
(a) Whether any perennial river is available near the
command area or not ?
(b) If an inundation river is available, the maximum
discharge of that river is to be determined from the
highest flood level mark.
(c) From various investigations, it is necessary to ascertain
whether the river will be able to meet the total water
requirement or not ?
(3) Selection of Probable Site for
Barrage or Dam
(a) The course of the river should be straight for a
distance of about 1 km both on the upstream and
downstream side of the site.
(b) The width of the river should be minimum and the
section of the river should be well defined.
(c) A suitable basin should be available for the storage
reservoir.
(d) The elevation of the site should be higher than that
of culturable command area.

(e) The storage reservoir should not submerge much


valuable land

(f) The capacity of the reservoir should fulfill the total


water requirement.
(4) Discharge Observation of the River
(5) Marking of Gross Command Area and
Culturable Area

When it is decided to take up the project, gross


command area and culturable areas should be
marked on the map.
(6) Deciding the Alignment of the Canals
(a) The alignment of the main canal should be taken such a way so that
unnecessary cutting and banking is avoided.
(b) The alignment of the main canal should be such that the branch
canals can be taken suitably to cover the whole culturable area.
(c) The alignment should cross the rivers, roads, railway lines
perpendicularly as much as possible.
(d) The alignment should not be taken through the valuable agricultural
land.

(e) The alignment should not pass through the thickly populated areas,
religious places and burial grounds.
(7) Final Survey
(8) Preparation of Drawing
(9) Office / Design Works
(10) Final Location Survey
(a) The centre line of the main and branch canals are
marked with concrete pillars with an interval of 30
m or 50 m

(b) The total land width required for the main and
branch canal should be marked with pillars at
suitable intervals.
IRRIGATION PROJECT REPORT
Important Irrigation Projects of India
(1) Bhakra Nangal Project
(2) Damodar Valley Project
(3) Farraka Barrage Project
(4) Gandak Project
(5) Godavari Barrage Project
(6) Hirakud Project
(7) Mahanadi Delta Project
(8) Mayurakshi Project
(9) Kansabati Project
(10) Kosi Project
(11) Tungavadra Project
(12) Ukai Project
Quality of Irrigation Water
Just as every water is not suitable for human
beings, in the same way, every water is not
suitable for plant life. Water containing
impurities, which are injurious to plant growth,
is not satisfactory for irrigation, and is called the
unsatisfactory water.
The various types of impurities which make the
water unfit for irrigation are classified as:
1) Sediment Concentration in water
2) Total concentration of soluble salts in water
3) Concentration of sodium ions compared to
other cations
4) Concentration of potentially toxic elements
present in water
5) Bicarbonate concentration as related to
concentration of Ca + Mg
6) Bacterial Concentration
(1) Sediment Concentration
The Sediment of fine silt improve the fertility of
land. Where as other types of sedimentation
decreases the fertility. Again excessive
sedimentation creates trouble in canal and
reservoirs. So water should not contain
excessive suspended sediment.
In general, ground water or surface water from
reservoirs etc does not have sufficient sediment
to cause any serious problems in irrigation
(2) Total concentration of soluble salts
Salts of calcium, magnesium, sodium and
potassium, present in the irrigation water may
prove injurious to plants. When present in
excessive quantities, they reduce the osmotic
activities of the plants, and may prevent
adequate aeration, causing injuries to plant
growth. The injurious effects of salts on the
plant growth depend upon the concentration of
slats left in the soil.
The salinity concentration of the soil solution
(Cs) after the Consumptive water (Cu) has been
extracted from the soil, is given by
The salt concentration is generally expressed by
ppm (parts per million or by mg/l, both units
being equal.
The critical salt concentration in the irrigation
water depends upon many factors, however,
amounts in excess of 700 ppm are harmful to
some plants and more than 2000 ppm are
injurious to all crops.
The salt concentration is also measured by
determining the electrical conductivity of water.
They are directly proportional to each other.
Based on the value, the irrigation water is
classified as given in the table:
(3) Concentration of sodium ions

If the concentration of sodium ions in respect of


other ions increases too much, then the soil
grains breaks down. The soil becomes less
permeable. It starts crusting when dry and its pH
increases towards that of alkaline soil. High
sodium soils are plastic, sticky when wet and are
prone to form clouds and they crust on drying
The proportion of sodium ions present in the
soils, is generally measured by a factor called
Sodium-Absorption Ration (SAR) and represents
sodium hazards of water. SAR is defined as:
The SAR value can be reduced by adding gypsum
(CaSO4) to the water or to the soil.
Example: What is the classification of irrigation
water having the following characteristics:
Concentration of Na, Ca and Mg are 22, 3 and
1.5 milli-equivalents per litre respectively, and
the electrical conductivity is 200 micro mhos per
cm at 25 degree C?

Hence, it is S2.
As electrical conductivity is 200 micro mhos per
cm at 25 degree C, it is C1
Hence it is S2-C1 type of water.
(4) Concentration of potentially toxic elements
A large number of elements such as boron, selenium
etc may be toxic to plants. Traces of Boron are essential
to plant growth but its concentration above 0.3 ppm
may prove toxic to certain plants.
Even for the most tolerant crops, boron concentration
should not exceed 4 ppm. Boron is generally present in
various soaps. The waste water containing soap should
therefore be used with great care in irrigation.

Selenium, even in low concentration is toxic and must


be avoided
(5) Bicarbonate concentration
High concentration of bi-carbonate ions may
result in precipitation of calcium and magnesium
bicarbonates from the soil solution, increasing
the relative proportion of sodium ions and
causing sodium hazards.
(6) Bacterial Concentration
The Bacterial contamination of irrigation water is
not directly harmful to the plants, but the food
grains or fruits which are grown by using
bacterial contaminated water may be harmful to
human beings.
Components of a surface irrigation system
Methods of Distribution of Water

Surface Subsurface
methods methods

Sprinkler Drip
irrigation irrigation
a) Surface Irrigation: Just flooding water. About 90% of
the irrigated areas in the world are by this method.

b) Sprinkler Irrigation: Applying water under pressure.


About 5 % of the irrigated areas are by this method.

c) Drip or Trickle Irrigation: Applying water slowly to the


soil ideally at the same rate with crop consumption.

d) Sub-Surface Irrigation: Flooding water underground


and allowing it to come up by capillarity to crop roots.
Drip
Irrigation
Surface Method
Water is applied to the field in either the controlled or
uncontrolled manner.

Controlled: Water is applied from the head ditch and


guided by corrugations, furrows, borders, or ridges.
Uncontrolled: Wild flooding.

Surface irrigation is entirely practiced where water is


abundant. The low initial cost of development is later offset
by high labour cost of applying water. There are deep
percolation, runoff and drainage problems
Border Irrigation System
❖ In a border irrigation, controlled surface flooding is
practiced whereby the field is divided up into strips by
parallel ridges or dykes and each strip is irrigated
separately by introducing water upstream and it
progressively covers the entire strip.

❖ Border irrigation is suited for crops that can withstand


flooding for a short time e.g. wheat.

❖ It can be used for all crops provided that the system is


designated to provide the needed water control for
irrigation of crops.

❖ It is suited to soils between extremely high and very low


infiltration rates.
Furrow Irrigation
▪ In furrow irrigation, only a part of the land
surface (the furrow) is wetted thus minimizing
evaporation loss.
▪ Furrow irrigation is adapted for row crops like
corn, banana, tobacco, and cabbage. It is also
good for grains.
▪ Irrigation can be by corrugation using small
irrigation streams.
▪ Furrow irrigation is adapted for irrigating on
various slopes except on steep ones because of
erosion and bank overflow.
Where Sprinkler Irrigation is suitable?
(1) When the land topography is irregular and
hence unsuitable for surface irrigation
(2) When the land soil is excessively permeable,
so as not to permit good water distribution
by surface irrigation
(3) When the land gradient is steeper and soil is
easily erodible
(4) When the watertable is high
(5) When the water is available with difficulty
and is scarce
Advantages of Sprinkler Irrigation
(1) Sprinkler irrigation can be efficiently used for a
wide range of topographic conditions, soils and
crops.
(2) With the use of sprinkler irrigation erosion of soil
can be controlled as surface runoff is eliminated
(3) A better control on irrigation can be enforced
and also light irrigation is possible which is required
for seedlings and plants which are very young.
(4) No land preparation is required thus labour cost
is reduced.
(5) Uniform application of water is possible
(6) Does not require borders, field channels etc
and hence more land is available for cropping.
(7) Even small streams of irrigation water can be
used efficiently.
(8) The time and amount of application of
fertilizer can be better controlled.
(9) The water application efficiency can be as
high as 80%
Disadvantages of Sprinkler Irrigation
(1) Wind may distort the sprinkler pattern, thus
resulting in non-uniform application of
irrigation water.
(2) The first investment involved in the sprinkler
irrigation method is high.
(3) A constant water supply is needed for the
most economical use of the equipment of the
sprinkler irrigation system
(4) Water must be clean and free from sand etc.,
to prevent clogging of the sprinkler

(5) The power requirement is high since


continuous pumping of water is required

(6) Heavy soils with poor intake can not be


irrigated efficiently.
Drip Irrigation
Drip or Trickle irrigation is
accomplished by using small
diameter plastic pipes with
drip nozzles commonly
Called drippers or emitter
to deliver water to the
land surface near the
base of the plants.
Water is delivered at a
very slow rate of about 2
to 10 liter per hour to
keep the moisture within the
desired range for plant growth
The advantages of drip irrigation over other
methods of irrigation is the excellent control of
water application which is provides.
The efficiency can be as high as 90%
The evaporation loss is minimal and deep
percolation loss almost avoided
Erosion of the soil will not happen
It is most suitable for saline water
The growth of the plants is maximum
The main drawbacks of this method are
Cost of drip irrigation is considerable higher than
other methods.

The problem of blockage of outlets exists

Some expertise is required for operation


Sub-Surface Irrigation
In this method, the water is applied to the root zone of
the crops by underground network of pipes. The
network consists of main pipe, sub-main pipes and
lateral perforated pipes. The perforated pipes allow
the water to drip out slowly. This method is suitable
for permeable soil like sandy soil and also where
evaporation loss is more.
CHOICE OF IRRIGATION METHODS
The following criteria should be considered

(a) Water supply available


(b) Topography of area to be irrigated
(c) Climate of the area
(d) Soils of the area
(e) Crops to be grown
(f) Economics
(g) Local traditions and skills
Water Requirement of Crops
Water requirements of a crop
Every crop requires a certain quantity of water
after a certain fixed interval, throughout its period
of growth.
If the natural rain is sufficient and timely so as to
satisfy both these requirements, no irrigation water
is required for raising that crop.
In England, for example, the natural rainfall
satisfies both these requirements for practically all
crops, and, therefore, irrigation is not significantly
needed in England.
But in a tropical country like India, the natural
rainfall is either insufficient, or the water does
not fall after fixed intervals, as required by the
crops.
Since the magnitude as well as the frequency of
the rainfall varies throughout a tropical country,
certain crop may require irrigation in certain part
of the country, and the same crop may not
require any irrigation in some other part of the
country.
The area where irrigation is a must for
agriculture is called the arid region, while the
area in which inferior crops can be grown
without irrigation is called a semi-arid region.
The term Water requirements of a crop means
“the total quantity and the way in which a crop
requires water, from the time it is sown to the
time it is harvested”.
water requirement varies with the crop as well as
with the place.
• In other words, “different crops will have
different water requirements and the same crop
may have different water requirements at
different places of the same country depending
upon the climate, type of soil, method of
cultivation, and useful rainfall, etc”.
Factors Affecting Water Requirement
(a) Water Table: If water table is nearer to
ground surface, water requirement is less.
(b) Climate: In hot climate evaporation loss will
be more hence water requirement will be
more.
(c) Ground Slope: If slope of the ground is steep,
the water flows down very quickly and the
soil gets little time to absorb requisite
moisture. So water requirement will be more.
(d) Intensity of Irrigation: If intensity of irrigation for a
particular crop is high, then more area comes under
the irrigation system and the water requirement will
be more.
(e) Type of Soil: In sandy soil, water percolates very
quickly and can not be retained. So, water
requirement is more.
(f) Method of Application of Water: In surface method,
more water is required to meet up the evaporation
loss. In sub-surface method, less water is required.
(g) Method of Ploughing: In deep ploughing (by
tractor) less water is required, because soil can retain
moisture for longer period.
Consumptive use of water
❖ The water needs of a crop thus consists of
transpiration and evaporation and is called
evapotranspiration or consumptive use.

❖ Evapotranspiration is dependent on climatic


conditions like temperature, daylight hours,
humidity, wind movement, type of crop, stage
of growth of crop, soil moisture depletion, and
other physical and chemical properties.
Determination of Consumptive Use
(A) Direct Measurement
Tank and Lysimeter methods
Field experimental plots
Soil moisture studies
Inflow and Outflow studies for large areas
(B) By Use of Equations
❖ Penman Method
❖ Blaney-Criddle method
❖ Hargreaves class A pan evaporation method
Blaney-Criddle Method
Assumes that main factors are Temperature and
Day-time hours.
Cu= k.f
here, k = Crop factor
f = Monthly consumptive use factor
= p/40 (1.8t + 32)
p = percent of day time hours of the year
t = Temperature in degree centrigade
Penmann Method
ETo = C x [W x Rn + (1 - W) x f(U) x (Es – Ea)]
where
ETo = Reference crop evapotranspiration in mm/day
C = Factor to compensate for the effect of day and night weather
condition
W = Temperature related weighting factor
Rn = Net radiation in mm/day = (1 – α) x Rs - Rnl
α = Reflection coefficient, (albedo) the numerical values of which for
most crops is 0.25
Rs = Incoming short wave radiation
= (0.25 + 0.5 x (SH / SHM) x Ra in mm/day
SH = Mean value of actual sunshine hours for the period considered
SHM = Maximum possible sunshine hours for the period considered
Ra = Extra terrestrial radiation in mm/day
Rnl = Net long wave radiations in mm/day
= F1 x F2 x F3
F1 = (2.01 x 10-9) x (T + 273)4
F2 = 0.34 – 0.044 √Ea
F3 = 0.1 + 0.9 x (SH / SHM)
f(U) = Wind related function
= 0.27 x (1 + U2 / 100)
Ea = Actual vapor pressure in mbar
= Es x RH
Es = Saturation vapor pressure in mbar
= 6.11 x exp [17.27 T / (237.3 + T)]
RH = Relative humidity
Crop Evapotranspiration, Cu or ETc = Kc X ETo
Typical Kc for different types of full grown crops
Determination of Irrigation Requirement of
Crops
In order to determine the irrigation requirement of a
certain crop, during its base period, the following
terms are used:

(a) Effective Rainfall (Re): Effective rainfall that part of


rainfall which is available to the plants

Effective Rainfall = % of Total Rainfall (Generally 80%)


• Effective Rainfall = 0.6 * Total Rainfall - 10 (Total
Rainfall < 70 mm)
• Effective Rainfall = 0.8 * Total Rainfall - 24 (Total
Rainfall > 70 mm)

(b) Consumptive Irrigation Requirement (CIR):


Consumptive irrigation requirement is defined as
the amount of irrigation water that is required to
meet the evapo-transpiration needs of the crops.
CIR = Cu - Re
(c) Net Irrigation Requirement (NIR): Net
irrigation requirement is defined as the
amount of irrigation water required to meet
the evapotranspiration as well as other needs
such as leaching.
NIR = CIR + leaching + water lost in deep
percolation etc
= Cu – Re + leaching + water lost in deep
percolation etc
(d) Field Irrigation Requirement (FIR): Field
irrigation requirement is the amount of water
required to meet NIR plus the water lost in the
process of field application.
if ηa is the water application efficiency, then
FIR = NIR / ηa

(e) Gross Irrigation Requirement (GIR): It is


the water required to satisfy FIR and
conveyance loss.
If ηc is the conveyance efficiency,
GIR = FIR / ηc
Table below gives the details for a certain crop. Using
Blaney – Criddle equation and a crop factor k = 0.75,
determine the following (i) Consumptive use (ii) CIR
(iii) FIR (iv) GIR. The latitude of the place is 30º N.
The water application efficiency is 0.7 and water
conveyance efficiency is 0.65.

Month Monthly avg Useful


temp ºC rainfall
Nov 19 -
Dec 19 1.2
Jan 12.5 0.8
Feb 13 -
Month Monthly avg Monthly % of day f = p/40
temp ºC (t) time hours (p) (1.8t + 32)

Nov 19 7.19 11.9


Dec 19 7.15 10.9
Jan 12.5 7.3 9.9
Feb 13 7.03 0.7
Cu = k *∑f = 0.75 * 42.4 = 31.8 cm
Re = 1.2 + 0.8 = 2.0 cm
CIR = Cu- Re = 31.8 – 2.0 = 29.8 cm
NIR = CIR
FIR = NIR / ηa = 29.8 / 0.7 = 42.6 cm
GIR = FIR / ηc = 42.6 / 0.65 = 65.5 cm
Water Resources
Engineering – II (CE 604)
Water Requirement of Crops
Asst. Prof. Ujjwal Saha
Department of Civil Engineering
IIEST, Shibpur
Crop Period or Base Period

The time period that elapses from the instant of its


sowing to the instant of its harvesting is called the
crop-period.

The time between the first watering of a crop at the time


of its sowing to its last watering before harvesting is
called the Base period.

Crop period is slightly more than the base period, but for
all practical purposes, they are taken as one and the same
thing, and generally expressed in B days.
Gross Command Area (G.C.A.) : The gross command
area is the total which can be commanded or irrigated
by a canal system.

Unculturable Area: The area where agriculture can not


be done and crops can not be grown is known as
unculturable area.

Culturable Command Area (C.C.A.): The total area


within an irrigation project where cultivation can be
done and crops can be grown is known as culturable
command area.

G.C.A.= C.C.A. + Unculturable area


The culturable command area can further be classified
as:

Culturable cultivated area: It is area where cultivation


has been done at present or at a particular time.

Culturable uncultivated area: It is that area where


cultivation is possible but it is not being cultivated at
present or at a particular time
Intensity of Irrigation: The intensity of irrigation may be
defined as a ratio of cultivated land for a particular
crop to the total culturable command area.

Root Zone Depth: Root zone depth is the maximum


depth in soil strata in which the crop spreads its root
system and derives water from the soil.

Crop Season: The period during which some particular


types of crops can be grown every year on the same
land is known as crop season
Kharif Season: this season range from June to October.
The crops are sown at the beginning of the monsoon
and harvested at the autumn. Example: Rice, Millet,
Maize, Jute, Groundnut

Rabi Season: This season ranges from October to


March. Rabi crops are- Wheat, Gram, Mustard, Onion,
Rapeseed, Linseed, Pulses.

Crop Ratio: it is the ratio of the area irrigated in Rabi


season to the area irrigated in Kharif season
Cash Crops: The crops are cultivated by the farmers to
sell in the market to meet their current financial
requirements is known as cash crops. The crops like
vegetable, fruits are considered as cash crops
Number of Watering
The initial watering which is done on the land to
provide moisture to the soil just before sowing any
crops is known as paleo or paleva.

The first watering is done when the crop has grown to


about three centimeters. This watering is known as
Kor watering and the period is known as Kor period.

Subsequently watering is done at regular intervals till


the crops attains maturity.
Time Factor: The ratio of number of days the canal has
actually kept open to the number of days the canal
was designed to kept open during the base period is
known as Time Factor

Capacity Factor: Generally, it is not required for the


canal to run to its maximum discharge capacity for all
the time. So, the ratio of average discharge to the
maximum discharge (design discharge) is known as
Capacity Factor
Water requirements of a crop
Every crop requires a certain quantity of water after a certain fixed interval,
throughout its period of growth.
If the natural rain is sufficient and timely so as to satisfy both these requirements,
no irrigation water is required for raising that crop.
In England, for example, the natural rainfall satisfies both these requirements for
practically all crops, and, therefore, irrigation is not significantly needed in
England.
But in a tropical country like India, the natural rainfall is either insufficient, or
the water does not fall after fixed intervals, as required by the crops.
Since the magnitude as well as the frequency of the rainfall varies throughout a
tropical country, certain crop may require irrigation in certain part of the country,
and the same crop may not require any irrigation in some other part of the
country.
The area where irrigation is a must for agriculture is called the
arid region, while the area in which inferior crops can be grown
without irrigation is called a semi-arid region.
The term Water requirements of a crop means “the total quantity
and the way in which a crop requires water, from the time it is
sown to the time it is harvested”.
water requirement varies with the crop as well as with the place.
In other words, “different crops will have different water
requirements and the same crop may have different water
requirements at different places of the same country depending
upon the climate, type of soil, method of cultivation, and useful
rainfall, etc”.
Factors Affecting Water Requirement
(a) Water Table: If water table is nearer to ground
surface, water requirement is less.
(b) Climate: In hot climate evaporation loss will be
more hence water requirement will be more.
(c) Ground Slope: If slope of the ground is steep, the
water flows down very quickly and the soil gets little
time to absorb requisite moisture. So water
requirement will be more.
(d) Intensity of Irrigation: If intensity of irrigation for a
particular crop is high, then more area comes under
the irrigation system and the water requirement will
be more.
(e) Type of Soil: In sandy soil, water percolates very
quickly and can not be retained. So, water
requirement is more.
(f) Method of Application of Water: In surface method,
more water is required to meet up the evaporation
loss. In sub-surface method, less water is required.
(g) Method of Ploughing: In deep ploughing (by
tractor) less water is required, because soil can retain
moisture for longer period.
Irrigation Efficiencies
Efficiency is the ratio of the water output to the water
input and is expressed as percentage.

Input minus output is nothing but losses, hence, if


losses are more, output will be less and, therefore
efficiency will be less.

Hence, Efficiency is inversely proportional to the


losses
Water is lost in irrigation during various processes and
therefore there are different kinds of irrigation
efficiencies.

(a) Water Conveyance Efficiencies (ηc)


Takes into account the conveyance loss or transit loss
It is the ratio of amount of water applied to the farm to
the amount of water supplied from the reservoir.
ηc = (Wf / Wr ) *100
Where ηc = water conveyance efficiency
Wf = water delivered to the field
Wr = water supplied from the reservoir

(b) Water Application Efficiency (ηa)


It is the ratio of the quantity of water stored into the
root zone of the crops to the quantity of water
delivered to the field
ηa = (Ws / Wf ) * 100
Where ηa = water application efficiency
Ws = water stored in the root zone
Wf = water delivered to the field

Wf = W s + R f + D f
Where Rf = Surface runoff from the field
Df = Deep percolation loss below the root
zone.
(c) Water Use Efficiency (ηu)
It is the ratio of water used beneficially to the amount
of water delivered.
ηc = (Wu / Wf) * 100
Wu= Water used beneficially
Wf = water delivered to the field
(d) Consumptive Use Efficiency (ηcu)
(e) Water Storage Efficiency (ηs)

The concept of water storage efficiency given an


idea of how much water has been stored in the
root zone during irrigation.
ηs = (Ws / Wn) * 100
Ws = Water stored in the root zone
Wn = Soil moisture deficiency = Field capacity
– Available Moisture
Consumptive use of water
❖ The water needs of a crop thus consists of
transpiration and evaporation and is called
evapotranspiration or consumptive use.

❖ Evapotranspiration is dependent on climatic


conditions like temperature, daylight hours, humidity,
wind movement, type of crop, stage of growth of
crop, soil moisture depletion, and other physical and
chemical properties.
Determination of Consumptive Use
(A) Direct Measurement

Tank and Lysimeter methods


Field experimental plots
Soil moisture studies
Inflow and Outflow studies for large areas

(B) By Use of Equations

❖ Penman Method
❖ Blaney-Criddle method
❖ Hargreaves class A pan evaporation method
Blaney-Criddle Method
Assumes that main factors are Temperature and Day-time hours.
Cu= k.f
Where, k = Crop factor
f = Monthly consumptive use factor
= p/40 (1.8t + 32)
p = percent of day time hours of the year
t = Temperature in degree centrigade
Penmann Method
ETo = C x [W x Rn + (1 - W) x f(U) x (Es – Ea)]
where
ETo = Reference crop evapotranspiration in mm/day
C = Factor to compensate for the effect of day and night weather
condition
W = Temperature related weighting factor
Rn = Net radiation in mm/day = (1 – α) x Rs - Rnl
α = Reflection coefficient, (albedo) the numerical values of which for
most crops is 0.25
Rs = Incoming short wave radiation
= (0.25 + 0.5 x (SH / SHM) x Ra in mm/day
SH = Mean value of actual sunshine hours for the period considered
SHM = Maximum possible sunshine hours for the period considered
Ra = Extra terrestrial radiation in mm/day
Rnl = Net long wave radiations in mm/day
= F1 x F2 x F3
F1 = (2.01 x 10-9) x (T + 273)4
F2 = 0.34 – 0.044 √Ea
F3 = 0.1 + 0.9 x (SH / SHM)
f(U) = Wind related function
= 0.27 x (1 + U2 / 100)
Ea = Actual vapor pressure in mbar
= Es x RH
Es = Saturation vapor pressure in mbar
= 6.11 x exp [17.27 T / (237.3 + T)]
RH = Relative humidity
Crop Evapotranspiration, Cu or ETc = Kc X ETo
Typical Kc for different types of full grown crops
Determination of Irrigation Requirement of
Crops
In order to determine the irrigation requirement of a
certain crop, during its base period, the following
terms are used:

(a) Effective Rainfall (Re): Effective rainfall that part of


rainfall which is available to the plants

Effective Rainfall = % of Total Rainfall (Generally 80%)


• Effective Rainfall = 0.6 * Total Rainfall - 10 (Total
Rainfall < 70 mm)
• Effective Rainfall = 0.8 * Total Rainfall - 24 (Total
Rainfall > 70 mm)

(b) Consumptive Irrigation Requirement (CIR):


Consumptive irrigation requirement is defined as
the amount of irrigation water that is required to
meet the evapo-transpiration needs of the crops.
CIR = Cu - Re
(c) Net Irrigation Requirement (NIR): Net irrigation
requirement is defined as the amount of irrigation
water required to meet the evapotranspiration as well
as other needs such as leaching.
NIR = CIR + leaching + water lost in deep percolation etc
= Cu – Re + leaching + water lost in deep
percolation etc
(d) Field Irrigation Requirement (FIR): Field irrigation
requirement is the amount of water required to meet
NIR plus the water lost in the process of field
application.
if ηa is the water application efficiency, then
FIR = NIR / ηa

(e) Gross Irrigation Requirement (GIR): It is the


water required to satisfy FIR and conveyance
loss.
If ηc is the conveyance efficiency,
GIR = FIR / ηc
Table below gives the details for a certain crop. Using
Blaney – Criddle equation and a crop factor k = 0.75,
determine the following (i) Consumptive use (ii) CIR
(iii) FIR (iv) GIR. The latitude of the place is 30º N. The
water application efficiency is 0.7 and water
conveyance efficiency is 0.65.

Month Monthly avg Useful


temp ºC rainfall
Nov 19 -
Dec 19 1.2
Jan 12.5 0.8
Feb 13 -
Month Monthly avg Monthly % of day f = p/40
temp ºC (t) time hours (p) (1.8t + 32)

Nov 19 7.19 11.9


Dec 19 7.15 10.9
Jan 12.5 7.3 9.9
Feb 13 7.03 0.7
Cu = k *∑f = 0.75 * 42.4 = 31.8 cm
Re = 1.2 + 0.8 = 2.0 cm
CIR = Cu- Re = 31.8 – 2.0 = 29.8 cm
NIR = CIR
FIR = NIR / ηa = 29.8 / 0.7 = 42.6 cm
GIR = FIR / ηc = 42.6 / 0.65 = 65.5 cm
Classes of Soil Water
Water present in the soil may be classified under
three heads:
1. Hygroscopic water: When an oven dried sample
is kept open in the atmosphere, it absorbs some
amount of water from the atmosphere. This is
known as hygroscopic water, and is not capable of
movement by the action of gravity or capillary
forces
2. Capillary water: Capillary water is that part, in
excess of hygroscopic water, which exists in the
pore space of the soil by molecular attraction.
3. Gravitational water: Gravitational water is that
part in excess of hygroscopic and capillary water
which will move out of the soil if favourable
drainage is provided.
Saturation capacity: When all the pores of the soil are filled
with water, the soil is said to be under saturation capacity or
maximum water-holding capacity. It is the upper limit of
possible moisture content. It can be determined if porosity is
known.

Field capacity: The field capacity of soil is the moisture


content of the soil after free drainage has removed most of the
gravity water.
At field capacity, the large soil pores are filled with air, the
micro pores are filled with water and any further drainage is
slow.
At field capacity, the water can be used by the plants.
Moisture equivalent:
This is an artificial moisture property of the soil and is
used as an index of the natural properties.
It is the percentage of moisture retained in a small
sample of wet soil 1 cm deep when subjected to a
centrifugal force 1000 times as great as gravity, usually
for a period of 30 minutes.
Moisture equivalent ≈ Field capacity
= 1.8 to 2 Permanent wilting point
= 2.7 Hygroscopic coefficient
Permanent Wilting Point
Permanent wilting point or the wilting coefficient is that
water content at which plants can no longer extract
sufficient water from the soil for its growth. It is at the
lower end of the available moisture range.
If the plant does not get sufficient water to meet its needs,
it will wilt permanently. A plant is considered to be
permanently wilted when it will not recover after being
placed in a saturated atmosphere.
As an approximation, the permanent wilting % can be
estimated by dividing the field capacity by a factor varying
from 2.0 to 2.4, depending upon the amount of silt in the
soil. For most of the soils, the wilting coefficient is about
150% of the hygroscopic water.
Available Moisture:
The difference in water content of the soil between field
capacity and permanent wilting is known as available
water or available moisture.

Readily Available Moisture:


It is that portion of the available moisture that is most
easily extracted by plants, and is approximately 75% of
the available moisture.
Soil Moisture Deficiency:
Soil moisture deficiency or field moisture deficiency is
the water required to bring the soil moisture content of
the soil to its field capacity.

Depth of water stored in root zone.


In order to estimate the depth of water stored in the
root zone of soil containing water up to field capacity,
let,
d = depth of root zone (in metres) ; Fc = field capacity
(expressed as ratio);
γ = unit weight of soil; and γw = unit weight of water.
Considering unit area (1 sq. metre) of soil area;
This depth of water will be available for
evapo-transpiration
Limiting Soil Moisture Conditions

• It is essential to
maintain readily
available water in the
soil if crops are to
make satisfactory
growth.
Depth and Frequency of Irrigation

At any time, the moisture content in the soil should be


between the field capacity and the optimum water
content, the optimum level upto which the soil
moisture may be allowed to be depleted without fall
in the yield
Irrigation watering is done upto the field capacity. Then
water is gradually utilized consumptively by the plants
and the soil moisture starts falling. When, the soil
moisture reaches the optimum value, fresh dose of
irrigation may be done so that the water content is
again raised to the field capacity.
If, d = depth of root zone (in metres) ; Fc = field
capacity (expressed as ratio); m0 = optimum moisture
content. γ = unit weight of soil; and γw = unit weight of
water.
Then, the depth of water dw to be given during each
watering is

If Cu is the daily consumptive use rate, frequency of


watering fw = dw / Cu (days)
Delta

Each crop requires a certain amount of water after a certain


fixed interval of time, throughout its period of growth.

The depth of water required every time, generally varies


from 5 to 10 cm depending upon the type of the crop.

If this depth of water is required five times during the base


period, then the total water required by the crop for its full
growth, will be 5 multiplied by each time depth. The final
figure will represent the total quantity of water required by
the crop for its full-fledged nourishment.
This total depth of water (in cm) required by a crop
to come to maturity is called its delta (∆).
or
Delta is the total depth of the water required by a
crop during the entire period the crop is in the field
and it is denoted by the symbol (∆).
Example 1
If rice requires about 10 cm depth of water at an average interval of about 10 days.
and the crop period for rice is 120 days, find out the delta for rice.
Solution.
Water is required at an interval of 10 days for a period of 120 days.
Hence, No. of required waterings = 120/10 = 12
Therefore, Total depth of water required = No. of waterings x Depth of watering
= 12 x 10 cm = 120 cm.
Hence, ∆ for rice =120 cm. Ans.

Example 2
If wheat requires about 7.5 cm of water after every 28 days, and the base period for
wheat is 140 days, find out the value of delta for wheat.
Solution.
No. of required waterings = 140/28 = 5
The depth of water required each time = 7.5 cm.
:. Total depth of water reqd. in 140 days = 5 x 7.5 cm = 37.5 cm
Hence, ∆ for wheat = 37.5 cm. Ans.
Average Approximate Values of ∆ for Certain Important Crops
Duty of Water.

The duty of water is the relationship between the


volume of water and the area of the crop it matures.
This volume of water is generally expressed as, “a unit
discharge flowing for a time equal to the base period of
the crop”.
If water flowing at a rate of 1 cubic metre per second,
runs continuously for B days, and matures 200 hectares,
then the duty of water for that particular crop will be
defined as 200 hectares per cumec to the base of B days.
Hence, duty is defined as the area irrigated per cumec of discharge
running for base period B. The duty is generally represented by the letter
D.
Or
Duty represents the irrigation capacity of unit water. It is the relation
between the area of the crop irrigated and the quantity of irrigation
water required during the entire period of that crop.
The duty varies with the place of its measurement because
of the continuous conveyance losses as water flows.
The duty of water goes on increasing as water flows.
The duty at the head of the water course is lesser than the
duty at the head of the field, and greater than the duty
at the head of the distributary.
Relation between Duty, Delta and Base period
Let, base period of the crop be B days, and
one cumec of water be applied to this crop on the field for B days.
Now, volume of water applied to this crop during B days
= V = (1 x 60 x 60 x 24 x B) m3
= 86,400 B m3
By definition of duty (D), one cubic meter supplied for B days matures D hectares
of land.
:. This quantity of water (V) matures D hectares of land or 104 D sq. m of area.
Total depth of water applied on this land
= Volume/area = 86400 B / 104 D = 8.64 B / D metres
By definition, this total depth of water is called delta (∆),
∆ = 8.64 B / D meter
∆ = 864 B / D cm
where, ∆ is in cm, B is in days ; and D is duty in hectares/cumec.
Example
Find the delta for a crop when its duty is 864
hectares/cumec on the field. The base period of this crop is
120 days.
Solution.
In this question, B = 120 days; and D = 864
hectares/cumec
Since, ∆ = 864 B / D cm
= 864 x 120 / 864
= 120 cm
Factors Affecting Duty
(1) Methods and systems of irrigation
Perennial vs inundation irrigation system
Flow irrigation vs lift irrigation

(2) Mode of applying water


The flood irrigation system has lesser duty than the furrow
system. Sub-surface irrigation system gives higher duty

(3) Methods of cultivation


If the land is properly ploughed and made quite loose before
irrigating, the soil will have high water retention capacity
in its unsaturated zone. Thus, duty will be increased.
(4) Time and frequency of tilling
Frequent tilling reduces the loss of moisture through
the weeds hence increasing duty
(5) Type of crop
(6) Climatic condition of the area
The climatic conditions which affects the duty are: (i)
Temperature (ii) wind (iii) humidity (iv) rainfall. High
wind and Temperature increases the evaporation
loss thus decreasing duty
(7) Base period of the crop
If the base period of the crop is more, the amount of
water required will be high, hence duty will be low.
(8) Quality of water
If harmful salts are present, more water need to be
applied to leach off the harmful salts, reducing the
duty. Fertilizers cause less consumption of water
hence increase duty
(9) Method of assessment for taxation
Volumetric assessment vs assessment based on area of
cultivation
(10) Canal condition
In an earthen canal, the seepage and percolation losses
will be high resulting in low duty.
(11) Character of soil and sub-soil of the canal
Coarse grained permeable soil vs fine grained soil
(12) Character of soil and sub-soil of the field
Coarse grained permeable soil vs fine grained soil
Methods of Improving Duty
(1) Suitable method of applying water to the root zone
depth of the water should be used so that various
losses is reduced.
(2) The land should be properly ploughed and levelled
before sowing the crop.
(3) The land should be cultivated frequently, since
frequent cultivation reduce loss of moisture
(4) The canals should be lined
(5) The water should be conveyed quickly, thus
reducing evaporation loss.
(6) The idle length of the canal should be reduced
(7) The alignment of canal in sandy soil should be
avoided
(8) The canal should be so aligned that the areas to be
cultivated are concentrated along it
(9) The source of supply of water should be such that it
gives good quality of water
(10) The rotation of crops must be practiced
(11) For taxation, Volumetric method of assessment
should be used
(12) The farmers must be trained in the proper use of
water so that they apply correct quantity of water at
correct timing
The left branch canal carrying a discharge of 20 cumecs
has a culturable command area of 20,000 hectares.
The intensity of Rabi crop is 80 % and the base period
is 120 days. The right branch canal carrying discharge
of 8 cumecs has culturable command area of 12,000
hectares, intensity of irrigation of Rabi crops is 50%
and the base period is 120 days. Compare the
efficiencies of the two canal systems in terms of duty.
For the left canal
Area under Rabi crop = 20000 x 0.8 = 16000 hectares,
Discharge = 20 cumecs
Duty = 16000/20 = 800 hectares/cumecs

For the right canal


Area under the Rabi crop = 12000 x 0.5 = 6000
hectares, Discharge = 8 cumecs
Duty = 6000/8 = 750 hectares/cumecs
The left canal system is more efficient which has higher duty
Numericals
Water Resources Engineering – II
(CE 604)

GATE Question

Asst. Prof. Ujjwal Saha


Department of Civil Engineering
IIEST, Shibpur
GATE 2021 (Set 2)
GATE 2020 (Set 1)
GATE 2020 (Set 2)
GATE 2019 (Set 2)
GATE 2018 (Set 2)
GATE 2017 (Set 2)
GATE 2015
Gram: duty = 8.64 X 18 / 0.12 = 1296 hec/cumec
Discharge reqd = 2000 x 0.3 / 1296 = 0.46 cumec

Wheat: duty = 8.64 X 18 / 0.15 = 1036.8 hec /cumec


Discharge reqd = 2000 x 0.5 / 1036.8 = 0.96 cumec

Total discharge (as both wheat and gram are rabi crops) =
0.46 +0.96 = 1.42 cumec
GATE 2015
Ans : (B)
GATE 2014
GATE 2013
Answer
Irrigation water requirement= 48 - 8 cm = 40 cm
=0.4 m
B=10 days
D=8.64 x 10/0.4
=216 hectares/cumec
GATE 2012
Answer :

Irrigation water requirement= 550 - 100 =450mm


=0.45m. Base period= 120 days.
Duty=(8.64 x 120/0.45) x 0.6 =1382.4 hectares/cumec
Area irrigated= 1382.4 x 0.01 =13.82 hectares
GATE 2010
Answer: Q no 50:
Irrigation requirment= 18 x 0.5 x 1 =9 cm
Area= 36 hectare. Water application efficiency=75%
Amount of water= (0.09 x 36 x 10^4 )/ 0.75
= 43200 m3
Frequency of irrigation= 9/0.5 = 18 days
Discharge required for the pump
= 43200/(18 x 10 x 60 x60)
= 0.0666 m3/s
= 67 L/s
Answer: Q no 51:
Moisture Required=18 x 0.8 x 0.5 =7.2 cm
Frequency of irrigation = 7.2/0.4 = 18 Days.
Quantity of water by running a pump for 10 h at
rate of 40 L/s for 18 days = 40 x 18 x 10 x 60 x 60 =
25920 x 10^3 L = 25920 m3.
Area irrigated= 25920 x 0.75/ 0.072 =27 x 10^4 m3
= 27 hectares.
GATE 2009
Answer:
Net water required= =105 - 15 =90 cm =0.9 m
Base period = 90 days.
Duty, D = 8.64 x 90 / 0.9
= 864 ha/cumec
GATE 2008
An outlet irrigates an area of 20ha. The
discharge at this outlet to meet the
evapotranspiration requirement of 20mm
occurring uniformly in 20 days neglecting other
field losses is
(A) 2.52 l/s (B) 2.31 l/s (C) 2.01 l/s (D) 1.52 l/s
Answer: (B)
Water required to meet evapo-transpiration = 20
x 10^4 x 20 x 10^-3 =4000 m3
Discharge required at outlet= 4000/(20 x 24 x 60
x 60) =0.00231 m3/s = 2.31 l/s

Duty = 8.64 x 20 / 0.02 = 8640 hec /cumec


Area = 20 hectares
Discharge = 20/8640 = 0.00231 m3/s
=2.31 l/s
GATE 2007
GATE 2007
Answer:
Area on which wheat is grown = 20,000 x 0.5 ha
B= 30 days. Delta= 120 mm =0.12 m
Duty= 8.64 x 30 /0.12
= 2160 ha/cumec
Outlet discharge = 20000 x 0.5/2160
=4.63 m3/s
GATE 2006
Answer: (c) 9 days
Readily Available moisture = (0.38 - 0.1)x .5 =0.14
Irrigation water requirement= 0.14 x 1000mm
(root depth)
= 140 mm
Frequency of irrigation= 140/15
= 9.33 days
GATE 2005
73. The culturable command area for a
distributary is 2 x 10^8 m2. The intensity of
irrigation for a crop is 40%. If kor water depth
and kor period for the crop are 14 cm and 4
weeks respectively, the peak demand
discharge is
(a) 2.63 m3/s (b) 4.63 m3/s (c) 8.58 m3/s
(d) 11.58 m3/s
Answer: (b) 4.63 m3/s

Duty= 8.64 x 28/.14 =1728 ha/cumec


Peak demand discharge= 2 x 10^8 x 0.4 x
10^(-4)/1728
= 4.63 m3/s
GATE 2004

Answer: (d) 0.7 cumec
Discharge reqd= sugarcane + wheat
= 0.36 + 0.27
= 0.63
Time factor is no of days the canal is actually run
Hence discharge reqd= 0.63/0.9
=0.7
GATE 2003
Calculate Duty
Then discharge. This is the minimum discharge
required as there are losses.
GATE 2003
Depth of water required to reach field capacity =
(0.15 - 0.05) x 1.1 = 0.11m = 110 mm
Consumptive use in 8 days=8 x 2.5 =20mm
So, Total water requirement= (110 + 20)/0.65
Water Resources Engineering – II
(CE 604)

Asst. Prof. Ujjwal Saha


Department of Civil Engineering
IIEST, Shibpur
OBJECTIVE QUESTIONS
IES 2020
IES 2019
(b)
IES 2019
IES 2019
IES 2017
(a)
IES 2016
(b)
IES 2015
• (d)
IES 2015
IES 2015
(a)
IES 2015
(b)
IES 2014
For an irrigated field having: field capacity = 30%,
permanent wilting point = 10%, permissible
depletion of available moisture =40%, dry weight
of soil=14.7 kN/m3, unit weight of water=9.8
kN/m3 and effective rainfall=30 mm, what is the
net irrigation requirement per meter depth of soil
? Dw=14.7*1/9.8 {0.4*(.3-.1)} *1000 dw-30
(a) 300 mm (b) 150 mm (c) 120 mm (d) 90 mm
Ans: (d)
IES 2014
A lift irrigation scheme using a discharge of 96 m3/hr
is planned to raise a crop with an average delta of
0.375 m. Intensity of irrigation is 60%. Assuming
3600 hours of working of the tubewell for a year, the
culturable command area is
(a) 96 Ha (b) 48 Ha (c) 154 Ha (d) 120 ha
Ans: (c )

Total volume of water applied = 96 x 3600 =


345600 m3
Area of crop can be irrigated = 345600/0.375 =
921600 m2 = 92.16 ha
Intensity of irrigation = 60 %
Hence CCA = 92.16 / 0.6 = 153.6 ha
IES 2014
A field measures 40 hectares. When 8 cumec of
water was supplied for 6 hours, 30 cm of water
was stored in the root zone. The field application
efficiency is nearly
(a) 70% (b) 80% (c)85 % (d) 90%
Ans : (a)

Water applied = 8 X 60 X 60 X 6 = 172800 m3


Depth of water applied = 172800 / 400000 =
0.432 m
Water stored = 0.3 m
Application efficiency = 0.3/0.432 x 100 %
=69.44 %
IES 2013
A rice crop is to be irrigated in a field covering an
area of 2400 ha, the duty and base period of rice
are given as 860 ha/cumec and 120 days
respectively. The volume of water required in
the field is nearly
(a) 500 ha.m (b) 1400 ha.m (c) 2000 ha.m
(d) 2880 ha.m
Ans: (d)

Calculate delta and multiply by area


IES 2013
In an irrigation project, in a certain year, 70%
and 46% of the culturable command area in
kharif and rabi, respectively, remained without
water and rest of the area got irrigation water.
The intensity of irrigation in that year for the
project was:
(a) 116 % (b) 84 % (c) 42% (d) 58%
Ans: (b)
IES 2013
The gross command area for a distributory is 5000
hectares, 80% of which is culturable irrigable. The
intensity of irrigation for rabi is 50 % and for kharif
is 30%, the average duty at the head of the
distributory is 2000 hectares/cumec for rabi
season and 900 hectares/cumec for kharif season.
The discharge required at the head of the
distributory from average demand considerations
is:
(a) 1 cumec (b) 1.33 cumec (c) 2.33 cumec
(d) 3.33 cumec
Ans: (b)
IES 2010
The discharge required for rabi and kharif crops
are 0.4 cumec and 0.3 cumec respectively The
capacity factor and time factors are 0.8 and 0.5
respectively, then what is the design discharge
for the distributary at its head.
(a) 0.8 cumec (b) 0.16 cumec (c) 1.0 cumec
(d) 1.24 cumec
Ans : (c )

The discharge of the canal will be maximum of


rabi and kharif = 0.4 cumec

After considering the time and capacity factor,


the discharge is = (0.4/0.8)/0.5 = 1 cumec
IES 2009
During a particular stage of the growth of a crop,
consumptive use of water is 2.2 mm/day. If the
amount of water available in the soil is 25% of
80 mm depth of water, what is the frequency of
irrigation
(a) 9 days (b) 13 days (c)21 days (d) 25 days
Ans: ( a)

Amount of water available = 0.25 x 80 = 20 mm


Eto = 2.2 mm/day
Frequency of irrigation = 20/2.2 = 9 days
IES 2009
In a canal irrigation project, 76 % of the
culturable command area (CCA) remained
without water during Kharif season and 58% of
CCA remained without water during rabi season
in a particular year. Rest of the areas got
irrigated in each crop respectively. What is the
intensity of irrigation for the project in that
year?
(a) 134 % (b) 76 % (c) 66 % (d) 58 %
Ans : (c )

Kharif season irrigated land = 100 – 76 = 24 %


Rabi season irrigated land = 100 – 58 = 42 %

Yearly intensity of irrigation = (24+42) =66 %


IES 2009
Consider the following statement:
Irrigation water has to be supplied to the crops
when the moisture level falls
1. Below Field capacity 2. to wilting point
3. Below wilting point
Which of the above statements is/are correct ?
(a) 1 only (b) 2 only (c) 3 only (d) 2 and 3
Ans : (a)
IES 2007
If the discharge required for different crops is
0.4 cumec in the field and the capacity factor
and time factors are 0.8 and 0.5 respectively,
then what is the design discharge for the
distributary at its head.
(a) 0.8 cumec (b) 0.16 cumec (c) 1.0 cumec
(d) 1.24 cumec
Ans: (c )

Discharge = (0.4 / 0.8) /0.5= 1 cumec


IES 2006
Consider the following terms relating to
irrigation requirements:
1) Consumptive irrigation requirement
2) Net irrigation requirement
3) Field irrigation requirement
4) Gross irrigation requirement
for a given set up, which one of the following is
the correct relation
(a) 1 > 2 > 3 > 4 (b) 1< 2 < 3 < 4
(c) (1 = 2) > 3 > 4 (d) 1 < (2 =3 ) < 4
Ans : (b)
IES 2004
The delta for a crop having a base period 120
days is 70 cm. what is the duty ?
(a) 2480 hectares/cumec
(b) 1481 hectares/cumec
(c) 148 hectares/cumec
(d) 1.481 hectares/cumec
Ans : (b)
IES 2004
For a culturable command area of 1000 hectares
with intensity of irrigation of 50%, the duty on
field for a certain crop is 2000 hectares/cumec.
What is the discharge required at head of water
course with 25% losses of water.
(a) 3/16 cumec (b) ¼ cumec (c) 1/3 cumec
(d) ½ cumec
Ans: (c )

Area to be irrigated = 1000 x 0.5 = 500 ha


Duty = 2000 ha/cumec
Discharge reqd = 500/2000 = ¼ m3/s
25 % is loss,
Hence discharge at the head of the water course
= (¼) / (¾) = 1/3 m3/s
IES 2004
What is the moisture depth available for
evapotranspiration in root zone of 1 m depth
soil, if dry weight of soil is 1.5 gm/cc, field
capacity is 30% and permanent wilting point is
10%
(a) 450 mm (b) 300 mm (c) 200 mm (d) 150 mm
Ans : (b)

Moisture available = 1.5 x (0.3 – 0.1) m


=0.3 m
= 300 mm
IES 2002
A canal was designed to supply the irrigation
needs of 1000ha of land growing rice of 140
days base period and having a delta of 130 cm. If
the canal water is used to irrigate wheat of base
period 119 days and having a delta of 50 cm, the
area that can be irrigated is:
(a) 452 ha (b) 904 ha (c) 1105 ha (d) 2210 ha
Ans: (d)

For Rice, Land = 1000 ha, B = 140 days, delta = 1.3 m


Duty = 8.64 X 140 / 1.3 = 930.47 ha/cumec
Discharge in the canal = 1000/930.47 = 1.07 m3/s

For wheat, B= 119 days, delta = 0.5 m


Duty = 8.64 x 119/0.5 = 2056.32 ha/cumec
Hence area can be irrigated = 2056.32 x 1.07 ha
= 2210.2 ha
IES 2001
10 m3/s of water is diverted to a 32 hectares
field for 4 hours. Soil proving after irrigation
showed that 0.3 m of water has been stored in
the root zone. Water application efficiency in
this case would be:
(a) 96% (b) 66.67 % (c) 48% (d) 24%
Ans: (b)

Water applied = 10 x 60 x 60 x 4 m3 = 144000 m3


Depth of water applied = 144000 / 32 X 10^4 m
=0.45 m
Water application efficiency = 0.3/0.45 x 100 %
=66.67 %
IES 2000
If the depth is 8.64 cm on a field over a base
period of 10 days, then the duty is
(a) 10 hectares/cumec
(b) 100 hectares/cumec
(c) 864 hectares/cumec
(d) 1000 hectares/cumec
Ans : (d)
IES 1997
The following data were recorded from an
irrigation field: Field capacity= 20%, permanent
wilting point= 10%, permissible depletion of
available soil moisture= 50%. Dry unit weight of
soil = 1500 kgf/m3, effective rainfall= 25mm.
Based on these data, the net irrigation
requirement per metre depth of soil will be
(a) 75 mm (b) 125 mm (c) 50 mm (d) 25 mm
Ans : (c )

Available moisture = field capacity – permanent


wilting point = 0.2 – 0.1 = 0.1
Moisture to be supplied by irrigation = 0.1 x 0.5 = 0.
05
Depth of water reqd = (1500/1000) x 0.05
= 0.075 m = 75 mm
Depth of irrigation water reqd = 75 – 25 = 50 mm
IES 1995
Given that the base period is 100 days and the
duty of the canal is 1000 hectares per cumec,
the depth of water will be
(a) 0.864 cm (b) 8.64 cm (c) 86.4 cm (d) 864 cm
Ans: (c) 86.4 cm
IES 1993
Application efficiency of irrigation is best defined
as the ratio of:
(a) Water for normal consumptive requirement
to water depleted from the root zone.
(b) Water actually stored in the root zone to
water delivered to the farm
(c) Water reaching the farm to water released
from the head works
(d) Water actually delivered to the farm to water
actually reaching the farm.
Ans : (b)
DESCRIPTIVE QUESTIONS
IES 1998
IES 1999
IES 2000
IES 2004
IES 2006
The irrigated area of a watercourse is 800 hectares.
The intensity of rice in this area is 65 %. The duration
for transplantation of rice crop is 15 days and the
total depth of water required by the crop is 60 cm, on
the field, during transplantation period. The useful
rain falling during the transplantation period is 15cm.
Find the duty of the irrigation water for the crop on
the field during transplantation at the head of the
field channel, if the loss of water in the water course
is 25%. Also find the discharge in the watercourse.
IES 2007
Determine the evapotranspiration and irrigation
requirement for wheat, if the water application
efficiency is 65 % and the consumptive use
coefficient for the growing season is 0.8 from
the following data
Month Mean monthly temp Monthly percentage of Effective rainfall
(oC) sunshine hour (cm)
November 18 7.2 2.6
December 15 7.15 2.8
January 13.5 7.3 3.5
February 14.5 7.1 2
IES 2008
From the data given in Table and taking kor
depth and kor period for wheat as 13.5 cm and 4
weeks respectively and for rice as 19 cm and 2.5
weeks respectively, compute the average
discharge requirement and peak demand
Crop Area under crop Total depth (cm) Base period (days) Average duty
(hectares) (ha/cumec)

Wheat 5000 37.5 140 3225

rice 2500 120 120 864


IES 2010
IES 2011
IES 2011
IES 2014
IES 2015
IES 2016
IES 2017
IES 2018
IES 2019
Flood
What is Flood?
Phenomenon when water goes out of
control and destroys villages and cities
with its strength called flood.
Causes of flooding
Physical HUMAN
• Urbanisation
• Climatological

• Nature of • Deforestation
drainage basin

• Coastal influences
Physical Causes:
• Climatological factors:

1. Intense rainfall (rainfall intensity > infiltration capacity) –


common in semi-arid areas and urban areas.

2. Prolonged period of rainfall produces saturation of the


soil and overland flow ( widespread flooding in UK
during summer 2007 was the result of the wettest
summer since 1766)

3. A sudden increase in temperature producing rapid


snowmelt, which can be made worse by frozen ground
limiting infiltration
Physical Causes:
• Nature of drainage basin:

1. Infiltration rate depends on rock and soil

2. Shape of the river basin, gradient and


vegetation covers affect how quickly
run-off reaches the channel
Physical Causes:
• Coastal influences:

• High seasonal tides,


storm surges, tropical
cyclones, Tsunamies can
result in higher water
levels in rivers and
low-lying coastal areas

– E.g. Bangladesh
Human causes of
flooding
1. Deforestation

– Reduces interception and


evapotranspiration

– Increase run-off

– Decrease in channel capacity


due to amount of sediment
being carried into river channel

– E.g. Maraca rainforest in the


Amazon Basin
2. Urbanisation increases the
magnitude and frequency of
floods by:
– The creation of
highly impermeable
surfaces such as
roads, roofs and
pavements

– Smooth surfaces
served by a dense
network of drains,
gutters and
underground sewers,
effectively increasing
drainage density
Other Human Causes
• Failure of hard engineering such as a dam
burst can produce catastrophic flooding
– Dam burst in an iron ore mine near Seville,
Spain 1999

• Land management techniques such as:


– drainage systems,
– digging ditches or
– ploughing up and down a slope
🡪 will decrease lagtime for water to reach the
river channel
Factors that increase the risk of flooding
Example Description Example Description
Impermeable Cutting
rock trees down
Steep slopes Very wet
soil
Hard dry soil Building in a
drainage
basin
Causes of Flooding

• Precipitation - Heavy rainfall over a long period creates


saturated soil = less infiltration = more surface run-off
• Flash Floods - Hot dry areas where the land is baked hard
causes flooding if there is an intense burst of heavy rainfall -
water cannot infiltrateBasically, anything
and results in rapid runoff.
• Snowmelt - When temperatures that rise and snow melts the
stored precipitation reduces
is released as runoff because infiltration
infiltration
will be low as the ground is still
andfrozen.
• Deforestation - Whenincreases
trees aresurface
cut down this reduces
interception, transpiration and storage by the trees. This
results in increased surfacerunoff
runoff.
• Urbanisation - When land is urbanised vegetation is removed
and the land is covered in concrete and tarmac. These
surfaces are impermeable = increase surface runoff.
Types of floods
• Coastal flooding
• Groundwater flooding
• River flooding
• Flash Flooding
• Sewer flooding
What are the
impacts of
flooding?
The table shows four groups of consequences of floods. Direct
consequences happen at the time of the floods. Indirect occur
later, after the floodwater has gone. Tangible damages can be
directly calculated by their economic value. Intangible damages
do not have a monetary value.
Tangible Intangible

Damage to houses, schools,


factories, roads, bridges, Loss of life, health effects,
Direct
furniture, cars, machinery, loss of ecological value.
power plants, water supply.

Loss of industrial
Inconveniences of recovery
production, traffic
Indirect after flood, increased
disruption, loss of
vulnerability of survivors.
customers.
Benefits of flooding
• Replenishes groundwater reserves

• Provides nutrient-rich sediment for agriculture


in the dry season

• Reduces the need for artificial fertilisers

• Flushes pollutants and pathogens away from


domestic areas

• May increase the fish supply


CAUSES OF NOTABLE FLOODS IN
INDIA: THE ANNUAL MONSOON
RAINS
A “HIMALAYAN TSUNAMII”
HIMALAYAN REGION OF
NORTH INDIA

June 24—JULY , 2013


Kedarnath, the home of a
deeply revered Hindu temple
visited by many pilgrims every
year, experienced the maximum
devastation.
KEDARNATH (Note: Temple in
Foreground)
PARTS OF NEW DELHI ALSO
FLOODED
IMPACTS
• At least 1,000 feared dead
• 70,000 evacuated
• More than 50,000 people cut off by
the waters
DEVASTATING FLOODS IN ASSAM STATE
AND IN NORTHEST INDIA

June 28—JULY 15, 2012


The Brahmaputra River
overflowed during monsoon
rains, flooding more than 2,000
villages and destroying homes in
the northeast of the country
FLOOD: ASSAM STATE;
JUNE 28, 2012
WILD BUFFALO GOING TO HIGHER GROUND;
JUNE 28
STRANDED IN NAELENI VILLAGE: JUNE 28
STRANDED
SOME OF THE 500,000 EVACUEES: JUNE 29
ELDERLY AND YOUNG EVACUEES: JUNE 29
HOMELESS BULUT VILLAGE FAMILY: JUNE 30
PUMPING DRINKING WATER: BULUT
VILLAGE; JUNE 30
2012: RECORD BREAKING IMPACTS

▪ Ninety-five dead
▪ Over 2 million homeless.
▪ Half a million evacuees are living in relief
camps with disease prone conditions
▪ Damaging landslides hindered relief
operations
INDIA

FLOOD PRONE AREA - 40 M.Ha.


ASSAM WEST BENGAL
BIHAR 9.40% 7.91% ORISSA
12.71% 4.18%

UTTAR PRADESH
21.90%

OTHER
STATES
43.9%
Predicting To reduce flood damage, we need to
FLOODS know the magnitude and frequency of
the floods a watershed is likely to get.
Flood Hazard Mitigation
Measures
FLOOD MANAGEMENT
APPROACH

• FLOODS CAN NOT BE ABSOLUTELY


CONTROLLED
• FLOODS CAN ONLY BE MANAGED TO
REDUCE FLOOD LOSSES
FLOOD MANAGEMENT
STRATEGIES

• MODIFY THE FLOODS


• MODIFY THE SUSCEPTIBILITY TO FLOOD
DAMAGE
• MODIFY THE LOSS BURDEN
• BEAR THE LOSS
FLOOD MANAGEMENT

ORGANISATIONS
• STATE FLOOD CONTROL DEPARTMENTS

• CENTRAL WATER COMMISSION

• GANGA FLOOD CONTROL COMMISSION

• BRAHMAPUTRA BOARD

• MHA - NDM DIVISION


FLOOD MANAGEMENTMEASURES

• STRUCTURAL : PHYSICAL WORKS FOR MEASURES


MODIFYING FLOOD MAGNITUDE
( TO KEEP FLOODS AWAY FROM PEOPLE )

• NON-STRUCTURAL : PLANNED ACTIVITY TO MODIFY


MEASURES SUSCEPTIBILITY TO FLOOD DAMAGE
( TO KEEP PEOPLE AWAY FROM
FLOODS )
FLOOD MANAGEMENT

STRUCTURAL MEASURES

• DAMS & RESERVOIRS


• EMBANKMENT
• CHANNEL IMPROVEMENT
• RIVER DIVERSION
• INTER BASIN TRANSFER
• ANTI EROSION WORKS
FLOOD MANAGEMENT
NON STRUCTURAL MEASURES

• FLOOD FORECASTING & WARNING

• FLOOD PLAIN ZONING

• FLOOD FIGHTING

• FLOOD INSURANCE

• RELIEF & REHABILITATION


What is a Floodplain ?

Floodplains are flat, alluvial tracts of land adjacent to


rivers defined by 100-year flood
FLOOD PLAIN ZONING

CONCEPT
• AN IMPORTANT NON-STRUCTURAL MEASURE.
• REGULATES LAND USE IN FLOOD PLAINS TO
RESTRICT DAMAGE BY FLOODS.
• INVOLVES DEMARCATION OF ZONES IN FLOOD
PLAINS COMPATIBLE WITH FLOOD RISKS
INVOLVED.
FLOOD PLAIN ZONING
WATER SUPPLY
III III II I
II
I
FACTORY

HOSPITAL
GOVT. OFFICE POWER HOUSE
PARK

RESIDENTIAL AREA UNIVERSITY


FREQUENT FLOOD
FLOOD ONCE IN 25 YEARS NORMAL CHANNEL
FLOOD ONCE IN 100 YEARS
FLOOD PLAIN ZONING

ZONE REGULATION
PRIORITY - I

ACTIVITY LIMITED TO WATER LEVELS


CORRESPONDING TO 100 YEARS FLOOD
FREQUENCY AND DRAINAGE CONGESTION FOR
50 YEARS RAINFALL.
FLOOD PLAIN ZONING

ZONE REGULATION
PRIORITY - II

ACTIVITY LIMITED TO LEVELS CORRESPONDING


TO
25 YEARS FLOOD FREQUENCY AND DRAINAGE
CONGESTION FOR 10 YEARS RAINFALL
FREQUENCY.
FLOOD PLAIN ZONING

ZONE REGULATION

PRIORITY - III

LESS ECONOMIC AND COMMUNITY ACTIVITY IN


AREAS VULNERABLE TO FREQUENT FLOODS.
FLOOD PLAIN ZONING

WARNING RESTRICTIVE PROHIBITIVE RESTRICTIVE WARNING


I II III II I

RIVER CHANNEL
INDIA
FLOOD FORECASTING

BEGINNING
• STARTED BY CENTRAL WATER COMMISSION

• YEAR 1958

• RIVER YAMUNA

• FORECASTING STATION DELHI RAILWAY BRIDGE


FLOOD FORECASTING AND WARNING
STEPS

■ DATA COLLECTION

■ DATA TRANSMISSION

■ DATA ANALYSIS & FORECAST FORMULATION

■ DISSEMINATION OF FORECAST
FLOOD FORECASTING
DATA COLLECTION
• HYDROLOGICAL
RIVER WATER LEVEL
RIVER DISCHARGE

• HYDROMETEOROLGICAL
RAINFALL
OTHER PRECIPITATION
eg. SNOW, HAIL ETC.
FLOOD FORECASTING

DATA TRANSMISSION

• WIRELESS ■ SATELLITE

• TELEPHONE ■ TELEGRAPH

• FAX ■ TELEX
FLOOD FORECASTING
DATA ANALYSIS & FORECAST FORMULATION

• CORRELATION

• MATHEMATICAL MODELS
FLOOD FORECASTING
DISSEMINATION

CIVIL AUTHORITY

FORECAST OTHER AGENCIES

PRESS, A I R
DOORDARSHAN

OTHER USERS
LOCAL
REPRESENTATIVES
FLOOD FORECASTING AND WARNING
PRESENT SYSTEM - SCHEMATIC DIAGRAM
RIVER STAGE & RIVER STAGE &
DISCHARGE AT DISCHARGE AT
WEATHER FORECASTS BASE STATION FORECASTING STATIONS RAINFALL

REGIONAL OFFICES REGIONAL OFFICES


OF I.M.D. OF I.M.D.

F.M.O OF I.M.D FLOOD FORECASTING F.M. O. OF I.M.D


CENTRES & CONTROL ROOMS

FORMULATION OF
C.E., S.E. CWC HQ MINISTRY OF WR
FORECAST

CIVIL AUTHORITIES
PRESS RADIO STATE GOVT.
ENGG. AUTHORITIES
P&T DOORDARSHAN CONTROL ROOM
DEFENCE INDUSTRIES
POLICE FLOOD COMMITTEE
RAILWAYS
FLOOD MANAGEMENT

scope

• Use of Remote Sensing and GIS


• Research and Development
• Weather / Rainfall Forecasting
• FF Modelling
• Capacity Building
• International Cooperation
INDIA

FLOOD FORECASTING NETWORK

• COVERS MAJOR AND INTER STATE RIVER


BASINS.

166 STATIONS :- INFLOW 27 & STAGE 139


INDIA

FLOOD FORECASTING STATIONS


85

27
17 15
9 8
3 2

GANGA B’PUTRA GODAVARI WEST EASTERN KRISHNA MAHANADI BARAK


FLOWING RIVERS
INDIA

FLOOD FORECASTING STATIONS


35
32

23

15 14
11 11

6
4 4 3 2 2 2 1

UP R AP B. A

A
A RA ND
A

AT

DE CT
AM SS

MP
W.
AN TTAR

NH
IH

AN
K

AL

LH
B S R I T A

N
TA

S JA R SH KH

RY
CH
A O D&
GU
A

A
U

’R R

HA
RN

A
M JH
KA
FLOOD MANAGEMENT ACTIVITIES

During flood

Pre flood Post flood


Stormwater Management
Basic Aim is to reduce or delay storm runoff or
carry all the stormwater safely to an
appropriate place.
Hard (Structural) measures include ponds,
swales or wetlands or a network of pipes
Soft measures also called LID (Low Impact
Development) includes large flat roofs, porous
parking lots, increase recharge, planting a high
delaying grass, increase forest cover, detention
basins, grassed waterways, porous sidewalks,
roof top gardens, fountain storage
Bio-retention Cells are depressions that contain vegetation
grown in an engineered soil mixture placed above a gravel
drainage bed. They provide storage, infiltration and
evaporation of both direct rainfall and runoff captured from surrounding
areas. Rain gardens, street planters, and green roofs are all variations of
bio-retention cells.

Infiltration Trenches are narrow ditches filled with gravel that intercept runoff
from upslope impervious areas. They provide storage volume and
additional time for captured runoff to infiltrate the native soil
below.

Continuous Porous Pavement systems are excavated areas filled with gravel
and paved over with a porous concrete or asphalt mix. Normally all
rainfall will immediately pass through the pavement into the gravel
storage layer below it where it can infiltrate at natural rates into the
site's native soil. Block Paver systems consist of impervious paver blocks
placed on a sand or pea gravel bed with a gravel storage layer
below. Rainfall is captured in the open spaces between the
blocks and conveyed to the storage zone and native soil below.
Rain Barrels (or Cisterns) are containers that collect roof
runoff during storm events and can
either release or re-use the rainwater during dry
periods.

Vegetative Swales are channels or depressed areas with


sloping sides covered with grass and
other vegetation. They slow down the conveyance of
collected runoff and allow it more time
to infiltrate the native soil beneath it.
Control and Training of Rivers

River Training, in its wider aspects, covers all those


engineering works which are constructed on a
river, so as to guide and confine the flow to the
river channel, and to control and regulate the
river bed configuration, thus ensuring safe and
effective disposal of floods and sediment loads.
Objectives of River Training
• To prevent the river from changing its course and
to avoid outflanking of structures like bridges,
weirs, aqueducts etc.
• To prevent flooding of the surrounding areas by
providing a safe passage of the flood waters
without overtopping the bank.
• To protect the river banks by deflecting the river
away from the attacked banks.
• To ensure effective disposal of sediment load.
• To provide minimum water depth required for
navigation.
Classification of River Training
(1) High Water training or training for discharge:
main aim to control of flood.

(2) Low water training or training for depth: primary


purpose is to provide sufficient water depth in
navigable channels during low water periods.

(3) Mean water training or training for sediment:


Aims at efficient disposal of suspended load and
bed load, thus to preserve the channel in good
shape.
Methods of River Training
The following are the generally adopted
methods:
1) Marginal embankments ( Dyke or Levee)
2) Guide Banks
3) Groynes and Spurs
4) Bank Protection
5) Pitched Island
1. Marginal Embankment
The marginal embankment or dyke or levee is an
earthen embankment of trapezoidal section
constructed approximately parallel to the bank of
the river to confine the flood water within a
section between the embankments.
the height depends on H.F.L.
Top width varies from 2m to 3m and the side slope
from 2:1 to 3:1 depending on the soil character.
Timber piles (sal ballah) are driven about 1 m away
from the toe at an interval of 1m ccentre to
centre. This is to stabilise the soil and to resist the
scouring effect of river.
The piles are kept projected about 50 cm above
the ground level. The in between space is
packed with boulder.
The river side is protected by stone pitching with
cement grouting.
The country side is
Protected by turfing.
The H.G.L is considered
And minimum of 50 cm
Cover is provided.
2. Guide Bank
When a barrage or dam is constructed across a
river which flows through the alluvial soil, the
guide banks must be constructed on both the
approaches to protect the structure from
erosion. It is a earthen embankment with
curved heads on both the ends.
The guide bank serves the following purposes:

(a) It protects the barrage from the effect of


scouring and erosion.
(b) It provides a straight approach towards the
barrage.
(c) It controls the tendency of changing the course
of the river.
(d) It controls the velocity of flow near the structure.
3. Groynes and Spurs
These are embankment type structures,
constructed transverse to the river flow,
extending from the bank to the river flow.
Therefore they may also be called as
‘Transverse Dykes’. They are constructed in
order to protect the bank from which they are
extended, by deflecting the current away from
the bank.
SPURS
Spurs are temporary structures and Groynes are
Permanent structures.

Spurs are projected from the river bank towards


the bed making angles 60º to 75º with the
bank of the river.
Bamboo Spur
Timber Spur
Boulder Spur:
GROYNES
The function of groynes is similar to that of spur. But
these are permanent structures constructed on
the curve of a river to protect the river bank from
erosion.
These are provided in series throughout the affected
length of the river bank.
The spacing between adjacent groynes is generally
kept as 2L where L is the length of the groynes.
These are recommended for the river where the
permanent solution of erosion control is
extremely necessary.
Different types of groynes
4. Bank Protection
(1) Brick Pitching : Bamboo or timber piles of length 3m
are driven at 15 cm centre to centre along a line about
1m away from the toe of the embankment.
Cement concrete (1:3:6)
of thickness 15 cm is laid
over a brick flat soling
on the space between toe
and pile line. The sloping
Side is protected by double
Layer brick pitching with
Cement mortar (1:6)
(2) Stone Riprap:
(3) Boulder Pitching:
(4) Concrete Slab Lining:
Slabs of size 50 cm * 50 cm * 10 cm are used.
(5) Sand Bag Protection (Temporary)
5. Pitched Island
Reclamation of Land
Flood Frequency Analysis
Predicting To reduce flood damage, we need to
FLOODS know the magnitude and frequency of
the floods a watershed is likely to get.
Flood Frequency Analysis

In the design of practically all hydraulic


structures the flood peak that can be expected
within the design life of the structure is of
primary importance to adequately proportion
the structure to accommodate its effect.
Flood and Runoff estimation
methods

Indirect Direct Methods


Methods
(measurements)
(Equations)
Current meter Direct Staff gauge
Methods

Crest stage gauge


Flow measurements
Flow measurements
Flow measurements
current meters
Rainfall-Runoff model

CN (SCS) Rational

Indirect Methods

Khosla’s’s Formula Unit Hydrograph

Strange’s Table
Estimation of Flood Magnitude

In case of a rural catchment, we model the


rainfall-runoff process and from the known
rainfall, we calculate the runoff. Some of the
models are HEC-HMS, VIC, SWAT etc.
FREQUENCY ANALYSIS
• Basic Problem:
To relate the magnitude of extreme events to
their frequency of occurrence through the use
of probability distributions.
Flood Frequency Analysis

• Statistical Methods to evaluate probability


exceeding a particular outcome - P (X >20,000
cfs)

• Used to determine return periods of rainfall


or flows
Probabilistic / Stochastic

Deterministic
Probability
• The probability P(A) of an event A is the relative
number of occurrences of event A after a very
large number of trials.
• It will be convenient to define the sum of the
probabilities of all the possibilities as one.
• Σi (P(Xi) = 1
• This means that a probability can range from zero
to one
• 0≤P≤1
Probability
Probability may be described as the relative frequency with which an event occurs
or will occur in the long run.
It is measured normally on a scale ranging from 0 (meaning impossible) to 1 (meaning certain)

ABSOLUTE CERTAINTY 1
That you will die some day
0.9
0.8
0.7 That you ( a man) will be taller than 1.65m.
0.6
Probability

0.5 That a coin will come down heads


0.4
scale

0.3
0.2
That you will win the lottery
0.1
ABSOLUTE IMPOSSIBILITY 0 That you can swim across the Atlantic
Probability Analysis
Histogram
0.24

0.2

0.16
Relative Frequency

0.12

0.08

0.04

0
450 500 550 600 650 700 750 800 850 900 950

Rainfall classes (mm)


Probability Distribution Function
Mean: μ 642 mm P(R < 642) = 0.5

Standard deviation: σ 110 mm P(R > 642) = 0.5


P(532 < R < 752) = 0.683
P(422 < R < 862) = 0.954

P(312 < R < 972) = 0.997


0.683

0.954

0.997

-330 -220 -110 μ =642 +110 +220 +330


Probability Analysis

Distribution Real data


50 occurrences (n=50)
Expected Actual

P(R < 642) = 0.5 25 29


P(R > 642) = 0.5 25 21

P(532 < R < 752) = 0.683 34 36

P(422 < R < 862) = 0.954 48 48

P(312 < R < 972) = 0.997 50 50


Cumulative Distribution Function
100

80
Cumulative %

60

40

20

450 500 550 600 650 700 750 800 850 900 950
Rainfall classes (mm)
Return Period
• Random variable:
• Threshold level:
• Extreme event occurs if:
• Recurrence interval:
• Return Period:
Average recurrence interval between events equaling or
exceeding a threshold
• If p is the probability of occurrence of an extreme
event, then

or
21
Relation between Probability and Return period
• What is the probability ‘p’ that in a certain period of time (i.e. 1 year),
the intensity of one event ‘X’ (rainfall) of duration ‘d’ (hours or days)
is bigger than ‘xt’ (100 mm)?
• P(X>100) when X is rainfall of i.e. 24 hs duration expressed in mm.
• We assume that the events are independents
• So we start observing events. You wait for (1 year) and you note
down the rainfalls in this year. For each year observation, there are
two possible outcomes
– “success" X ≥ xt (probability p)
– "failure" X < xt (probability 1-p)
Since the observations are independent, the probability of a
recurrence interval of duration T is the product of the probabilities
of (T-1) failures followed by one success:
First year: failure -> (1-p)
Second year: failure -> (1-p)
Third year: failure -> (1-p)
Forth year: Success!! -> p
So the probability of success in general is (1-p)*(1-p)*(1-p)*p=
(1-p)(t-1) * p
Assuming that the series of data is infinite, the E(T) can be
expressed as:

This is equal to:

So, the probability of occurrence of an event in any observation is


the inverse of its return period.
Annual Maximum Event
• It is the event whose magnitude is maximum
for the particular year.

• An annual maximum event has a return period


(or “recurrence interval”) of T years if its
magnitude is exceeded, on average, every T
years.
The T-year Flood
• We talked about an event, the T-year flood.
• For example, we talked about a 100-year flood
in Murshidabad
• The probability of the event T is 1/T
• For example, in any year, the chance of getting
the 100-year flood is 1/100 or 0.01
• The probability that the event T does not
occur is 1- 1/T or 0.99
Frequency Analysis by Plotting Position
Formula
Steps:
1. Annual maximum flood series data is prepared.
2. The series is arranged in decreasing order of
magnitude
3. The probability of an event being equal to or
exceeded is calculated by the plotting position
formula
P= m/ N +1
Where m is the order of the event and N= Total
number of events in the data
4. The return period or T is calculated as
T =1/P
5. The relationship of probability of occurrence
of various events and T is given by
Pr,n = nCrPrqn-r =( n! / (n - r)! r! ) Pr qn-r
Where P=1/T and q=1- P
6. Plot Q vs T
7. Fit best fit curve
8. Extrapolate to get other results for different T
values
Frequency Factors
• If the T is outside the ranges of the observed data,
Theoretical probability distribution is used
• Chow proposed using:
(1)
• where f (x)
X

29
EV-I (Gumbel) Distribution
The probability of occurrence of an event equal
to or larger than a value x0 is
(2)
In which y is a dimensionless variable given by
y= α (x-a) a =
Thus
(3)
Where = mean and σx = standard deviation
In Practice we require the value of X for a given P.
hence, we can write yp = -ln[-ln(1-P)]
And as P = 1/T

It follows from equ (1) and (3), The value of X with


a return period T is

where
Theoretically Gumbel’s assumption is valid for an
infinite sample size i.e. N ∞
But practically N is finite. Hence modification
based on sample size is required for
practical use.
Frequency Factors
• If the T is outside the ranges of the observed data,
Theoretical probability distribution is used
• Chow proposed using:
(1)
• where f (x)
X

33
EV-I (Gumbel) Distribution
The probability of occurrence of an event equal
to or larger than a value x0 is
(2)
In which y is a dimensionless variable given by
y= α (x-a) a =
Thus
(3)
Where = mean and σx = standard deviation
In Practice we require the value of X for a given P.
hence, we can write yp = -ln[-ln(1-P)]
And as P = 1/T

It follows from equ (1) and (3), The value of X with


a return period T is

where
Theoretically Gumbel’s assumption is valid for an
infinite sample size i.e. N ∞
But practically N is finite. Hence modification
based on sample size is required for
practical use.
Log-Pearson Type III Distribution
Steps:
1) Prepare the annual maximum flood data (x)
2) Take logarithm (base 10)
z= log(x)
3) The T year flood estimate is given by
4) From table calculate Kz based on T and Cs

T (years)
Cs 1 2 5 10 25 50 100 200
3 -0.667 -0.396 0.42 1.18 2.278 3.152 4.051 4.97
2.9 -0.69 -0.39 0.44 1.195 2.277 3.134 4.013 4.904
2.8 -0.714 -0.384 0.46 1.21 2.275 3.114 3.973 4.847
2.7 -0.74 -0.376 0.479 1.224 2.272 3.093 3.932 4.783
2.6 -0.769 -0.368 0.499 1.238 2.267 3.071 3.889 4.718
2.5 -0.799 -0.36 0.518 1.25 2.262 3.048 3.845 4.652
2.4 -0.832 -0.351 0.537 1.262 2.256 3.023 3.8 4.584
2.3 -0.867 -0.341 0.555 1.274 2.248 2.997 3.753 4.515
2.2 -0.905 -0.33 0.574 1.284 2.24 2.97 3.705 4.444
0.1 -2.252 -0.017 0.836 1.292 1.785 2.107 2.4 2.67
0 -2.326 0 0.842 1.282 1.751 2.054 2.326 2.576
5) After finding zT the corresponding value of xT
is obtained as xT =antilog(zT)
Log Normal Distribution
Special case of log-pearson type III distribution
where Cs=0
Procedures are same except the Kz values is
fixed and taken from the table as Cs=0
Confidence Interval
The Confidence interval indicates the limits
about the calculated value between which the
true value can be said to lie with a specific
probability based on sampling error

POPULATION
SAMPLE
For a confidence probability c, the confidence
interval of the flood is between values x1 and
x2. where x1 and x2 is given by :

Where f(c) = function of the confidence


probability c determined using the table of
normal variates as
C in 50 68 80 90 95 99
percentage
f(c) 0.674 1 1.282 1.645 1.96 2.58
Example:
Data covering a period of 92 years for the river
Anjana at Krishnanagar yielded mean and
standard deviation of the annual flood series
as 6437 and 2951 m3/s. using Gumbel’s
method estimate the flood discharge with a
return period of 500 years. What are the (a) 95
% and (b) 80 % confidence limit for the
estimate
For N =92 years, =0.5589 and Sn = 1.2020
Risk and
Reliability
The probability of at least one success in n years is
called the RISK.

Prob success = p = 1/T and Prob failure = q = 1-p

The probability of occurrence of an event r times in n


successive year is given by
RISK = 1 - P(0)
= 1 - Prob(no success in n years)
= 1 - (1-p)n
= 1 - (1 - 1/T)n

Reliability means that in the n years, the event


will not occur at all.

Reliability = (1 - 1/T)n
Risk
Example
What is the probability of at least one 50 yr flood in a
30 year period, where the probability of success in
any year is 1/T = 1/50 = 0.02

RISK = 1 - (1 - 1/T)n = 1 - (1 - 0.02)30

= 1 - (0.98)30 = 0.455 or 46%


If this is too large a risk, then increase design
level to the 100 year where p = 0.01

RISK = 1 - (0.99)30 = 0.26 or 26%


Safety Factor
Annual flood data of a river covering the period
1948 to 1979 yielded a mean flood of 29600
m3/s and a standard deviation of 14860 m3/s.
for a proposed bridge on this river, it is decided
to have an acceptable risk of 10% in its
expected life of 50 years. (i) Estimate the design
flood discharge by Gumbel’s method (ii) If the
actual flood value adopted in the design is
125000 m3/s, what are the safety factor and
safety margin.
For N=32 years, take yn = 0.5380 ans Sn = 1.1193
Design Flood
Design Flood: Flood adopted for the design of a
structure.
It depends on
▪ Type and importance of the structure
▪ Economic development of surrounding area
exp: Small structures such as culverts and storm
drains in rural areas can be designed for less
severe flood as the consequences of higher
than design flood may not be very serious.
For dam ?
SPILLWAY DESIGN FLOOD

Design flood used for the specific purpose of


designing the spillway of a storage structure. It
is the maximum discharge that can be passed
in hydraulic structure without any damage or
serious threat to the stability of the structure.
STANDARD PROJECT FLOOD (SPF)
The flood that would result from a severe
combination of meteorological and
hydrological factors that are reasonably
applicable to the region.

PROBABLE MAXIMUM FLOOD (PMF)


The extreme flood that is physically possible in a
region as a result of severemost combinations,
including rare combinations of meteorological
and hydrological factors
PMF is used in situations where the failure of
the structure would result in loss of life and
catastrophic damage and as such complete
security from potential flood is sought.

SPF is often used where the failure of a structure


would cause less severe damage

Typically SPF is about 40 to 60% of the PMF for


the same drainage basin
Design Storm (Standard Project Storm)
Duration
4. Abstract Initial and infiltration loss to get the
design rainfall hyetograph
Time from 0 6 12 18 24 30 36 42 48 54 60
start (h)

Cumulativ 0 15 24.1 30 34 37 39 40.5 41.3


e rainfall
(cm)

6-h UH 0 20 54 98 126 146 154 152 138 122 106


ordinates
(m3/s)

Time from 66 72 78 84 90 96 102 108 114 129 132


start (h)

6-h UH 92 79 64 52 40 30 20 14 10 6 0
ordinates
(m3/s)
Design Storm (Standard Project Storm)
Duration
4. Abstract Initial and infiltration loss to get the
design rainfall hyetograph
Time from 0 6 12 18 24 30 36 42 48 54 60
start (h)

Cumulativ 0 15 24.1 30 34 37 39 40.5 41.3


e rainfall
(cm)

6-h UH 0 20 54 98 126 146 154 152 138 122 106


ordinates
(m3/s)

Time from 66 72 78 84 90 96 102 108 114 129 132


start (h)

6-h UH 92 79 64 52 40 30 20 14 10 6 0
ordinates
(m3/s)
The ordinates of a mass curve of rainfall from a
severe storm in a catchment is given.
Ordinates of a 12-h unit hydrograph applicable
to the catchment are also given. Using the
given mass curve, develop a design storm to
estimate the design flood for the catchment.
Take φ index as 0.15 cm/h, estimate the
resulting flood hydrograph. Assume the
base flow to be 50 m3/s.
Tim Cum Incr 12-h First Desig Infiltr Rain DRH DRH DRH DRH Final Base Flood
e Rain eme UH arra n ation fall due of due to due to DRH Flow Hydro
(h) (cm) ntal (m3/ nge Stor loss Exce to 1.7 10.4 16.5 0.2 cm (m3 (m3/s graph
Rain s) men m (cm) ss cm cm cm rain /s) ) (m3/s)
(cm) t(cm (cm) (cm) Rain Rain Rain
)
0 0 0 0 0 0 0 0 0 0 0 0 50 50

12 12.2 12.2 32 3.5 1.8 1.7 54.4 0 0 0 54.4 50 104.4

24 30.5 18.3 96 2 12.2 1.8 10.4 163.2 332.8 0 0 496 50 546

36 34 3.5 130 18.3 18.3 1.8 16.5 221 998.4 528 0 1747 50 1797

48 36 2 126 12.2 2 1.8 0.2 214.2 1352 1584 6.4 3157 50 3207

60 98 3.5 166.6 1310 2145 19.2 3641 50 3691

72 75 127.5 1019 2079 26 3252 50 3302

84 50 85 780 1617 25.2 2507 50 2557

96 30 51 520 1238 19.6 1828 50 1878

108 15 25.5 312 825 15 1178 50 1228

120 7 11.9 156 495 10 672.9 50 722.9

132 0 0 72.8 247.5 6 326.3 50 376.3

144 0 0 115.5 3 118.5 50 168.5

156 0 0 0 1.4 1.4 50 51.4


(1) The probability that a 100 year flood may not
occur at all during the 50 year life of project
is …..

(2) The probability of a flood, equal to or greater


than 1000 year flood, occuring next year is
….

(3) A hydraulic structure with a life of 30 years is


designed for a 30 years flood. The risk of
failure of the structure during its life is …..
Flood Routing
Flood Routing Q

t
• Procedure to
determine the flood
Q
hydrograph at a
point on a
watershed from a
t
known hydrograph
upstream Q
• As the hydrograph
travels, it
– attenuates t
– gets delayed Q

t
Applications of routing techniques:
• Flood predictions
• Evaluation of flood control measures
• Assessment of effects of urbanization
• Flood warning
• Spillway design for dams
• Reservoir design
Routing methods can be broadly classified into
two categories :
i) Hydrologic routing
• Methods combine the continuity equation with
some relationship between storage and
outflow or possibly inflow.
• These relationships are usually assumed, or
empirical.
• An example of such a relationship might be a
stage-discharge relationship.
ii) Hydraulic Routing

• Hydraulic routing methods combine the


continuity equation with some more physical
relationship describing the actual physics of the
movement of the water.
• The momentum equation is the common
relationship employed.
• In hydraulic routing analysis, it is intended that
the dynamics of the water or flood wave
movement be more accurately described
Basic Equation of Hydrologic Routing
I = inflow rate
Q = outflow rate
S= storage
For a small time interval ∆t, the difference
between the total inflow volume and total
outflow volume in a reach is equal to the
change in storage in that reach

Where = average inflow in time

= average outflow in time

= change in storage
Numerical Equivalent

I 1 + I 2 – Q 1 + Q2 S 2 – S1
=
2 2 Δt
The time interval ∆t should be sufficiently
short so that the inflow and outflow
hydrographs can be assumed to be
straight lines in that time interval.
∆t should be shorter than the time of transit
of the flood wave through the reach
Hydrologic Storage Routing or
Reservoir Routing or Level Pool Routing

Storage or Reservoir routing is also called the


level pool routing as the water surface in the
reservoir is assumed to be horizontal.
Problem Definition
1) A flood wave I(t) is entering a reservoir provided with
an outlet such as spillway
2) The outflow is a function of the reservoir elevation
only or Q =Q(h)
As we know the flow over a weir is Q=CLH3/2
Where
Q = Discharge at the outlet
C = Discharge coefficient of weir
L = Length of crest
H = Depth above spillway
3) The storage in the reservoir is a function of
the reservoir elevation. Hint: storage =volume
volume=area*height. If area is fixed, then
storage S = S(h)

4) Due to passage of the flood wave in the


reservoir, the water level in the reservoir
changes with time, h=h(t). It depends on
inflow rate I(t)
Hence, storage and discharge change with
time
The Aim

To find the variation of S, h and Q


with time i.e. find S=S(t), Q=Q(t)
and h=h(t) given I=I(t)
Data Required

▪ Storage volume vs elevation for the reservoir


▪ Water-surface elevation vs outflow
Hence storage vs outflow discharge
▪ Inflow hydrograph, I = I(t) and
▪ Initial values of S, I and Q at time t=0
Methods
Variety of methods but all of them uses the
basic equation in various rearranged manners

Two commonly used semi-graphical methods


are discussed: Modified Pul’s method and
Goodrich Method
Modified Pul’s Method
Steps
1) From the known storage elevation and
discharge elevation data, prepare a curve of

2) On the same plot prepare a curve of Outflow


discharge (Q) vs Elevation (h)
3) The initial values of S, h and Q is noted. Also
Inflow hydrograph I(t) is given
4) Calculate the value of

And use equation below to calculate

5) From the plot in step 1, water surface elevation is


found out corresponding to
6) From the plot in step 2, the outflow discharge
Q2 is found out.

7) Deducting Q2∆t from gives

8) Use the value of inflow and

In the next time step to get the outflow of the


next time step
9) Repeat the procedure
Goodrich Method
Steps
1) From the known storage elevation and
discharge elevation data, prepare a curve of

2) On the same plot prepare a curve of Outflow


discharge (Q) vs Elevation (h)
3) The initial values of S, h and Q is noted also
Inflow hydrograph I(t) is given
4) Using the equation

Calculate the value of

5) From the plot in step 1, water surface


elevation is found out corresponding to
6) From the plot in step 2, the outflow discharge
Q2 is found out.

7) Deducting 2Q2 from gives

8) Use the new value of inflow and


In the next time step
9) Repeat the procedure
Hydrologic Channel Routing
In Reservoir routing, the storage S is a unique
function of the outflow discharge Q or S=f(Q)
In Channel routing the storage is a function of
both outflow and inflow discharges i.e.
S=f(I,Q)
Considering a channel reach having a flood flow,
the total volume in storage can be considered
under two categories as: Prism storage and
Wedge storage
PRISM STORAGE:
It is volume that would exist if the uniform flow
occurred at the downstream depth, i.e. the
volume formed by an imaginary plane parallel
to the channel bottom drawn at the outflow
section water surface
WEDGE STORAGE:
It is the wedge like volume formed between the
actual water surface profile and the top
surface of the prism storage
Wedge and
Prism Storage

• Positive wedge I>Q


• Maximum S when I = Q
• Negative wedge I<Q
Hydrologic river routing (Muskingum
Method)

Advancing
Flood
Wave
I>Q

K = travel time of peak through the reach


X = weight on inflow versus outflow (0 ≤ X ≤ 0.5)
X = 0 🡺 Reservoir, storage depends on outflow, no
wedge
X = 0.0 - 0.3 🡺 Natural stream

Receding
Flood
Wave
Q>I
Combine and rearrange

Simplified into the routing equation:

Subscript 1 refers to t1and 2 to t2 = (t+Δt)


Need K and Δt in the
same units
Need K and Δt in the
same units
Estimation of K and x
1) From the known inflow and outflow graph,
calculate change in storage at different time
using the continuity equation i.e.
∆S=∆t(I-Q)/2
2) Calculate the cumulative storage at each time S=
∑∆S
3) Assume any value of x (x should be between 0
and 0.5)
4) Calculate [xI + (1-x)Q ] values
5) Plot S vs [xI + (1-x)Q ] for several values of x
5) The value of x which gives straight line plot is
selected

6) The inverse slope of the straight line will be


the value of K i.e. K=x/y or S/ [xI + (1-x)Q ]
Routing Procedures
1) Estimate the values of K and x
2) Select appropriate value of ∆t. the value of ∆t
should lie within K and 2Kx or K>∆t > 2Kx
3) Calculate C0, C1 and C2 by these equations
4) Initially I1, Q1 and I2 is known
5) Q2 is calculated using the equation

6) In the next time step, Q3 is calculated as

7) Following the same procedure, calculate all


the outflows (Q)
Canal Irrigation
Introduction

A canal is an artificial channel constructed to carry water from a river or from a


tank or reservoir for various purposes such as irrigation, power generation,
navigation etc. An irrigation canal is the one which carries water from the source
to the agricultural fields for irrigation.
Classification of Irrigation canals

Based on Nature of Source of Supply


Permanent canals
A permanent canal is the one which is fed by a permanent source of supply.
It is a well graded channel and is provided with permanent regulation and
distribution works.
Perennial canals
Non-pernennial canals
Inundation canals
The canal gets its supplies only when the level in the river from which it
takes off rises during floods.
Not provided with any headworks for diversion of river water but obtain
their supplies through open cuts in the bank of the river.
Classification of Irrigation canals

Based on Function of Canal


Feeder Canal
A feeder canal is the one which is constructed only to feed another canal.
No direct irrigation is carried out from a feeder canal.
Indira Gandhi feeder canal 216 km long
Carrier canal
The canal carries water to feed another canal besides beign used for direct
irrigation.
Upper Chenab Canal in West Punjab (Pakistan)
Classification of Irrigation canals

Based on discharge and relative importance in a given network of canals


Main Canal
Branch Canal
Major Distributary
Minor Distributary
Water Course (Field Channel)
Classification of Irrigation canals

Main Canal
Principal canal of a network of irrigation canals.
Takes off directly from a river or reservoir or from the tail end of a feeder canal.
Usually a large capacity canal which supplies water to branch canals and major distributaries.
Branch Canal
These irrigation canals take off from the main canal on either side.
Generally carry a discharge over 5 cumecs.
Main function is to supply water to major and minor distributaries.
From small ones, direct irrigation is done.
Major Distributary
These are the irrigation canals which take off from the branch canals and sometimes from
the main canal.
Carry a discharge varying from 0.25 to 5 cumec.
Generally used for direct irrigation and supply water through outlets to water courses.
Minor Distributary
Take off from major distributaries and branch canals
Carry discharge less than 0.25 cumec
Water Courses or Field Channels
Small channels which carry water from the outlets of a major or minor distributary or a
branch canal to the fields to be irrigated.
Owned, constructed, controlled and maintained by the cultivators
Based on Canal Alignment
Ridge canal
It is aligned along the ridge or the natural watershed line
When the canal runs on a watershed, it can irrigate areas on both sides and hence a large
area can be brought under cultivation
No drainage can intersect a ridge line, so cross drainage works are not required to be
provided.
Economical
Contour canal
Aligned nearly parallels to the contour of the country
Irrigate areas only on one side
No cross drainage
Side slope canal
Aligned at right angles to the contours of the country
Neither on the watershed nor on the valley but in between the two along the slope
Parallel to the natural drainge
Irrigate areas on only one side
Steep bed slope
Based on Financial Output
Productive canal
When fully developed yields enough revenue to cover up its running cost and a net saving at
the rate of more than six percent of the capital invested initially for its construction
Protective canal
It is constructed as a relief work during famine to provide employment to the inhabitants of
the famine affected area and to protect that area against famine in future
Revenue from this canal usually does not cover its running charges and also does not repay
its initial capital expenditure.
Based on Soil through which it is constructed
Alluvial canals
Constructed through alluvial soils
Readily scoured and deposited
Available in Indo-Gangetic Plains in Northern India
Non alluvial canals
Those canals are constructed through hard soils or disintegrated rocks
Usually found in Central and Southern India
Based on Lining being Provided or not
Unlined canals
An unlined canal is the one which has its bed and banks made up of natural soil through
which it is constructed and not provided with a lining of impervious material
Velocity of flow in these canals is kept low so that bed and banks of the canal may not be
scoured.
Lined canals
Provided with a lining of impervious material on its bed and banks to prevent the seepage
of water
Higher velocities of flow can be allowed in lined canals and hence their cross-sectional areas
can be reduced.
Figure:
Figure:
Figure:
Design of Lined Canal
Introduction

Conservation of water supplies is important as the demand continues to increase and new sources
of supply are becoming scarce. Reduction in the amount of water lost through seepage suring
conveyance in canals is the most important principle of conservation. Seepage losses from unlined
canal system not only entail sizeable wastage of investment made on the construction of dam to
impound water and canal ssytem to distribute it, but also calls for further investment to control
the waterlogging in the irrigated areas.In certain cases, lining may be required purely from
technical considerations. For example, a canal cosntructed partly on rock and partly on permeable
foundation may be unsafe unless it is lined.Sometimes a hard lined surface is required to withstand
the high velocities. Apart from this, the engineer is required to produce a good economic
justification for the capital that is likely to be invested on lining.To justify the lining, economics of
canal lining must be worked out.
Advantages of Lining

Lining of channels reduces the seepage loss thus resulting in a saving of water which can be
utilised for irrigating additional area. The cost of irrigating per unit area is therefore reduced.
It is an important anti waterlogging measure as it reduces the percolation of water to the
ground water reservoir and thus prevents rising of water table.
The lining provides a smooth surface having a low value of rugosity coefficient. Hence a lined
canal has a higher velocity of flow.
Due to higher velocity, greater discharge can pass through a smaller area of cross-section of the
channel.
The higher velocity of flow in a lined canal results in reducing the evaporation loss.
The problem of silting is reduced.
Due to smaller cross-section, there is saving in cost of earthwork, land width and various
structures.
Lining prevents weed growth.
Lining prevalents the water flowing in the channel to come in contact with harmful salts which
might be present in the soil of the bed and banks of the channel
Reduces maintenance cost and possibility of breaching due to increased stability of channel
section.
Provides a stable channel section which is easier to operate.
Disadvantages of Lining

The lining of irrigatin channels involve a high initial investment.


It is difficult to repair the damaged lining.
A lined channel section is without a berm. As such, the additional safety provided by berms for
vehicular and pedestrian traffic is absent in case of lined channels.
Economics of Canal Lining

The lining of channel will be economically justified if the extra cost of providing the lining is less
than or equal to the benefits resulting from it.Thus the maximum rate of expenditure on lining
that is economically justifiable can be found out as follows:
Let C = cost of lining in rupees per square meter including the additional cost of dresssing the
banks for lining and accounting for the saving, if any resulting from the smaller cross-sections and
hence smaller area of land, quantity of earthwork and structures required for the lined sections.
This saving will be available on new canals excavated to lined cross-sections from the beginning,
but not on lining of existing unlined canals.
s and S = seepage losses in unlined and lined canals respectively in cubic metres per square meter
of wetted surface per day of 24 hours
p and P = wetted perimeter in metres of unlined and lined canals respectively
T = Total perimeter of lining in metres
d = number of running days of the channel per year
W = value of water saved in rupees per cubic metre
L = Length of the channel in metres
Y = life of lining in years
M = annual saving in rupees in operation and maintenance due to lining
B = annual estimated value in rupees of other benefits for the length of canal under consideration
which include prevention of waterlogging, reduced cost of drainage for adjoining lands etc.
X = percent rate of interest
Economics of Canal Lining

The value of water lost by seepage from the unlined section = pLsdW rupees
The annual saving in value of water otherwise lost by seepage
= (pLsdW −PLSdW )rupees = [LdW (ps − PS)]rupees
Total annual benefits resulting from the lining of channel

= [LdW (ps − PS) + B + M] rupees

Let a = [LdW (ps − PS) + B + M]


Additional capital expenditure on the construction of lined canal = TLC rupees
This cost must be recovered from the saving during the useful life of the lining Y years. If the
prevalent rate of interest is X, the net present worth (NPW) of the total annual benefits a over
the life of the lining (Y years) is determined from the following formula:

(1 + X )Y − 1
NPW = a
X (1 + X )Y

For the lining to be economically feasible, the additional initial cost of the lined canal (TLC)
should be equal to or less than the net present worth (NPW) of the annual benefits, i.e.,

TLC ≤ NPW
Example

An unlined canal giving a seepage loss of 4.0 cumec per million square metres of wetted area is
proposed to be lined with 12 cm thick cement concrete lining which costs Rs. 20.00 per square
meter. Work out the economics of lining and show if the scheme is justified on the basis of the
following data:
Annual revenue from crops = Rs. 4,00,000 per cumec of water
Discharge of the channel = 100 cumec
Area of the channel = 50 m2
Wetted perimeter of the channel = 22.4 m
Wetted perimeter of the lining = 22 m
Annual maintenance cost of unlined channel = 12 paise per m2
Seepage loss in lined channel = 0.01 cumec per million square meter of wetted perimeter
Saving in annual maintenance as a consequence to lining = 40%
Life of lining = 40 years
Rate of interest = 6 percent
Assume additional suitable data, if required.
Solution

Let L be the length of the canal.


The value of water lost per year by seepage from the unlined
section= (22.4 × L) × 1046 × 400000 = 35.84L rupees
The value of water lost per year by seepage from the lined
section= (22 × L) × 0.01
106
×400000 = 0.088L rupees
∴The annual saving in value of water otherwise lost by seepage= (35.84 − 0.084)L = 35.752L rupees
Annual saving in rupees in operation and maintenance due to lining
12 40
= 100 × 100 ×22.4 × L = 1.0752L rupees
∴Total annual benefits resullting from the lining of canal = (35.752L + 1.0752L)rupees = 36.8272L rupees
The net present worth (NPW) of the total annual benefits is given by
" #
40
(1 + 0.06) − 1
NPW = 36.8272L = 554.08 rupees
0.06 (1 + 0.06)40
Assuming total perimeter T of lining to be about 15% more than the wetted perimeter of the
lined canalT = 22 × 1.15 = 25.3m
Thus additional capital expenditure on the construction of the lined channel is

TLC = 25.3 × L × 20 = 506L rupees

Since NPW > TLC, the lining is economically justified.


Types of Lining

Exposed and hard surface linings


Cement concrete lining
Precast concrete lining
Cement mortar lining
Hydraulic lime concrete lining
Brick tile lining
Stone blocks lining
Asphaltic concrete lining
Buried membrane linings
Prefabricated asphaltic membrane lining
Polyethylene asphaltic membrane lining
Bentonite and clay membrane lining
Earth lining
Thin compacted earth lining
Thick compacted earth lining
Stabilized soil lining
Soil-cement lining
Types of lining

Figure:
Concrete lining

Concrete lining can be placed in many ways


hand placing by plastering on sides and bed
using forms and pouring alternate panels
using prefabricated concrete elements
When the lining is placed using the alternate panel method, guide forms are used.
Sections are poured alternately, with the finished sections being used as forms for
the sections in between.Small openings or expansion joints spaced at intervals of
1.5 to 3 m are needed for the expansion and contraction of non-reinforced
concrete. These joints are filled with flexible, asphaltic material to prevent water
leakage. For small canals, prefabricated concrete elements can also be used.
Figure:
Figure:
Figure:
Concrete block, brick or stone masonry lining

The concrete blocks, bricks or stones are laid flat on the compacted sides and
bed of the trapezoidal canal. The joints are filled with cement mortar, which
should have a cement-tosand ratio of 1:3 to 1:4 . A rectangular canal can be
constructed with a concrete or masonry bed and vertical masonry walls. Usually
the water side of the masonry structure is plastered, particularly if the bricks are
not of good quality.
Concrete block, brick or stone masonry lining
Design of Lined Canal

In case of channels lined with hard surfaced materials, two types of sections are used. A triangular
section with circular bottom is adopted for smaller channels and a trapezoidal section with
rounded corners is adopted for large channels. IS: 10430-1982 recommend only trapezoidal section
with rounded corners for all discharges. Typical cross-section of the lined channel is shown in the
figure. The various considerations for the design are given below.
Side Slope
It should be such that they are nearly equal to the angle of repose of the natural soil in the
subgrade so that no earth pressure is exerted over the back of the lining.
Freeboard

Type of channel Discharge(cumec) Freeboard(m)


Main and Branch canals >10 0.75
Branch canals and major distributaries 5 to 10 0.60
Major distributaries 1 to 5 0.50
Minor distributaries <1 0.30
Water courses < 0.06 0.10 - 0.15

Freeboard is measured from the full supply level to the top of the lining.
Figure:
Steps for Design

The following data is required


Discharge Q
Rugosity coefficient N
Bed Slope S
Side slopes
Limiting Velocity V
Steps
Compute hydraulic mean radius R from V, N and S using Manning’s equation as
 3/2
VN
R=
S 1/2
Using continuity equation, find area of cross-section
Q
A=
V
Calculate wetted perimeter
A
P=
R
For trapezoidal sections with rounded corners
A = BD + D 2 (θ + cotθ)

P = B + 2D(θ + cotθ)
Example

Design a trapezidal shaped concrete lined channel to carry a discharge of 200 cumec at a slope of
30 cm/km. The side slopes of the channel are 1.5:1. The value of N may be taken as 0.017.
Assume limiting velocity in the channel as 2 m/s.
Solution

From Manning’s equation, hydraulic mean radius R is given by


 3/2
VN
R=
S 1/2
V = 2 m/s, N = 0.017 and S = 3×10−4
h i3/2
2×0.017
∴R = 1/2 = 2.75 m
(3×10−4)
For side slope 1.5:1,
cotθ = 1.5
∴ θ = 0.588 radians
Area of cross-section A = VQ = 200
2 = 100 m
2

Wetted perimeter P = RA = 2.75


100
= 36.36 m

A = BD + D 2 (θ + cotθ)
or

100 = BD + D 2 (0.588 + 1.5)


or
100 = BD + 2.088D 2 (1)

P = B + 2D(θ + cotθ)
or
36.36 = B + 4.176D (2)
Solution(Contd.)

From equation (2), we get

100 = (36.36 − 4.176D)D + 2.088D 2


or
2.088D 2 − 36.36D + 100 = 0
or
D 2 − 17.41D + 47.89 = 0

D = 3.43 m
Then
B = 36.36 − (4.176 × 3.43) = 22.04 m
Thus Bed width = 22.04 m and depth of flow = 3.43 m
Design of Unlined Alluvial Channels
Introduction

A river in alluvium carries huge quantity of sediments along with water. This
sediment comprises soil particles of various sizes ranging from fine silt to coarse
sand. Sediments are either carried in suspension or get deposited on the river bed.
A portion of this sediment is also received by the irrigation canals which draw
water directly from the river. In case of unlined alluvial canals, the quantity of
silt transported may vary from section to section due to scouring of bed and sides
of the canal, as well as silting (deposition of silt) at any section. If the velocity of
flow in a canal is high, then bed and sides of a canal will be scoured. On the
other hand, silting may take place on bed and sides if velocity of flow is low.
Both silting and scouring change the cross section of the canal. Unlined alluvial
canals should be designed for such a velocity that no silting and no scouring
would occur. This velocity is known as non-silting and non-scouring velocity.
Regime Method

Introduction
Cross section of a stable alluvial canal would depend on the flow rate, sediment transport rate and
the sediment size. One method of designing an alluvial canal section is based of the regime
approach in which a set of empirical equations is used. These equations have been obtained by
analyzing the data of stable field channels.
Regime methods for the design of stable channels were first developed by the British engineers
working for canal irrigation in India in the nineteenth century. At that time problem of sediment
deposition was one of the major problems of canal design in India. In order to find a solution to
this problem, some of the British engineers studied the behaviour of such stretches of the existing
canals where the bed was in a state of stable equilibrium.
Regime Method

These stable reaches had not required any sediment clearance for several years of canal operation.
Such canals were called regime canals. These canals generally carried a sediment load smaller that
500 ppm.
Suitable relationships for the velocity of flow in regime canals were evolved. These relationships
are known as regime equations which find acceptance in other parts of the world as well.
The regime relations do not account for the sediment load and hence, should be considered valid
when the sediment load is not large.
Two widely accepted regime method of canal design are Kennedy’s method and Lacey’s method.
Kennedy’s Method

R. G. Kennedy (Executive Engineer, Irrigation Department, Punjab) collected data from 22 canals
of Upper Bari Doab canal system in Punjab. His observations in this canal system led him to
conclude that the sediment in a canal is kept in suspension solely by the vertical component of the
eddies which are generated on the channel bed. In his opinion, the eddies generating on the sides
of the canal had horizontal movements for greater part and therefore did not have sediment
transport power.
This means the sediment supporting power of a canal is proportional to its width and not wetted
perimeter.
Kennedy’s Method

Kennedy’s Equation
On plotting the observed data, Kennedy obtained the following relation, known as Kennedy’s
equation.
V0 = 0.55D0.64
Kennedy termed V0 as the critical velocity (in m/s), defined as the mean velocity which do not
allow scouring or silting in a canal having depth of flow equal to D (in m). This equation is
obviously applicable to such channels which have the same type of sediment as was presented in
the Upper Bari Doab canal system.
On recognizing the effect of the sediment size on the critical velocity, Kennedy modified the above
equation, V = 0.55mD0.64 in which m is the critical velocity ratio, and is equal to V/V0 Here V is
the critical velocity for all sizes of sediment while V0 is the critical velocity for the Upper Bari
Doab sediment.
Kennedy did not try to establish any other relationship for the slope of regime canals in terms of
either the critical velocity or the depth of flow. He suggested the use of Kutter’s equation
alongwith Mannig’s roughness coefficient. The final result do not differ much if one uses the
Manning’s equation instead of Kutter’s equation. Thus the equations

V = 0.55mD 0.64 (1)

Q = AV (2)

1
V = R 2/3S 1/2 (3)
n

enable one to determine the unknowns B, D and V for Given Q, n and m, if the longitudinal slope
Steps for design

Case 1
Given Discharge, critical velocity ratio, Manning’s n, Bed slope(1H:2V), Side slope
Assume a trial depth D.
Calculate velocity from equation 1.

V = 0.55mD 0.64

Compute cross-sectional area from A = Q/V.


Calculate B from

A = BD + D 2 /2

Calculate wetted perimeter



P =B +D 5

Calculate Hydraulic Radius


R = A/P

Calculate Velocity from equation 3.


This velocity should match with Keneddy’s velocity . Otherwise, go for next trial.
Example 1

Design an irrigation canal carrying a discharge of 30 m3 /s with critical velocity ratio m and
Manning’s n equal to 1.0 and 0.0225 respectively. Assume bed slope as 1 in 5000 and side slopes
as 1H:2V.
Solution

Trial D V0 A B P R V
1 2.0000 0.8571 35.0025 16.5 20.97 1.669 0.8859
2 2.5 0.9887 30.3443 10.89 16.48 1.842 0.9462
3 2.25 0.9242 32.4610 13.3 18.33 1.771 0.9217
4 2.23 0.9189 32.6470 13.52 18.51 1.764 0.9192
5 2.22 0.9163 32.7411 13.64 18.6 1.76 0.9180
6 2.221 0.9165 32.7316 13.63 18.59 1.76 0.9181
7 2.222 0.9168 32.7222 13.62 18.58 1.761 0.9182
8 2.223 0.9171 32.7128 13.6 18.57 1.761 0.9184
9 2.224 0.9173 32.7034 13.59 18.57 1.761 0.9185

Depth of channel = 2.223 m, Width of Channel = 13.6m

Figure:
Steps
Case 2
Given Discharge, critical velocity ratio, Manning’s n, B/D ratio.
Given B/D = c
Express A like this

A = BD + D 2 /2

A = cD 2 + D 2 /2 = D 2 (c + 0.5)

V = 0.55mD 0.64

Calculate D from this equation

Q = AV

or
Q = D 2 (c + 0.5)(0.55mD 0.64)

D = (Q/(0.55m(c + 0.5))1/2.64

Calculate B as B = cD
Calculate Hydraulic Radius
Example 2

Design an irrigation canal to carry a discharge of 5 m3/s. Assume Manning’s n = 0.0225, critical
velocity ratio m=1 and B/D = 3.24.
Solution

B/D = 3.24

D = (5/0.55(3.24 + 0.5))1/2.64 = 1.4m

B = 3.24D = 4.536m

A = (3.24 + 0.5)1.42 = 7.3304m2


P = B + D 5 = 7.6664m

R = A/P = 0.9561

V = 0.55D 0.64 = 0.6821m/s

n2 V 2
S = 4/3 = 1 in 4089
R
Lacey’s Theory

Gerald Lacey carried out a detailed study of the problem of designing stable channels in alluvium
and made significant contribution. He developed the regime theory and evolved a number of
equations on the basis of his own observations and the observations of the earlier investigators.
According to Lacey the width, depth and bed slope of a channel constructed in easily erodible
alluvium and carrying a fixed discharge and silt load are uniquely determined by nature. In other
words, for a channel constructed in alluvium to carry discharge, the width, depth and bed slope
will undergo modification by silting and scouring till equilibrium is attained. The channel is termed
as regime channel.
Lacey’s regime equations

Lacey considered hydraulic mean radius R as a more pertinent variable rather than depth of flow D
considered by Keneddy. Further as a measure of silt grade Lacey introduced a silt factor f with its
value equal to unity for the silt of channels of Upper Bari Doab Canal System. Lacey obtained a
relationship which is as follows
r
2
V = fR (4)
5
where V = mean velocity of flow in m/s
f = silt factor
R = hydraulic mean radius in m
This was the first regime equation given by Lacey.

For determining the channel dimension second equation would be necessary giving either
cross-sectional area A or perimeter P. Lacey on the basis of plotted data evolved a relationship
between A and V which is as follows

Af 2 = 140V 5 (5)
Lacey’s regime equation

Silt factor-Grain Size Relation

According to Lacey, the silt factor f is dependent on the mean particle size of the boundary
material in the channel. The value of f may be determined by the following relation given by Lacey

f = 1.76 m (6)
where m = mean particle size in mm = d50in mm
Lacey’s regime equation

V-Q-f Relation

From Continuity Equation,


Q = AV
∴Qf 2 = AVf 2

or
Qf 2 = 140V 6 (7)
or 1/6
Qf 2

V = (8)
140
where Q = discharge in cumec
Lacey’s regime equation

P-Q Relation
By combining equations 4 and equation 7, a relationship between P and Q can be obtained as follows.
Raising both sides of the equation 4 to the fourth power, we get
4 2 2
V4 = f R
25
Eliminating f2 between this equation and equation (7), we get

25V 4
 
Q = 140V 6
4R 2

or
25Q
= 140V 2
4R 2

Since
R = A/P

25Q
= 140V 2
4(A/P) 2

or
Q 560 2
= V
(A/P)2 25
or
560
P 2Q = (AV )2
25
or

P 2 = 22.4Q
or
p
P = 4.75 Q (9)
Regime flow equation

After determining the dimensions of a regime channel, the bed slope of the channel is required to
be determined. Thus by plotting a large number of data Lacey obtained the following flow
equation

V = 10.8 R 2/3S 1/3 (10)


where S = bed slope
It is called General Regime Equation which does not involve coefficient of rugosity as it is
considered that rugosity is implicit in R and S.
Regime Slope Equations

S-f-R Relation

From the equation of equation (10), we get

V 3 = 1260 R 2 S
From equation (4), we get

V 3 = (2/5)3/2f 3/2R 3/2


Hence from the above two equations,

1260 R 2 S = (2/5)3/2f 3/2R 3/2


or
f 3/2
S= (11)
4980R 1/2
S-f-Q Relation

Equating the values of V given by equations (4) and (8), we get


r  2 1/6
2 Qf
fR =
5 140
or  1/2  2 1/6
5 Qf
R 1/2 =
2f 140
 1/6
1/2 Q
∴R =
8.96f
Substituing this value of R in equation (11), we get
1/6
f 3/2 8.96f

S=
4980 Q
or
f 5/3
S= (12)
3340 Q 1/6
Design of Channel by Lacey’s Method

For the design of channel by Lacey’s method, the following equations are used:

f = 1.76 m

1/6
Qf 2

V =
140

Q = AV


P = 4.75 Q
r
2
V = fR
5
f 5/3
S=
3340Q 1/6
The items which must be known are Discharge Q, Silt factor f for mean particle size m
Steps for Design

Calculate the silt factor f from the 1st Equation if mean particle size is given
Calculate the velocity from the equation
1/6
Qf 2

V =
140

Calculate area form the equation as


Q
A=
V

Compute perimeter P from the P-Q relation


Knowing A and P and assuming side slopes of the channel as 1 horizontal to 2 vertical, determine bed width B and depth D from the following
relations

D2
A = BD +
2


P=B+ 5D

Calculate hydraulic mean radius R as


A
R=
P

Also calculate hydraulic mean radius R from equation as


5 V2
R=
2 f

Both the values of R obtained in the above two steps should be same.
Calculate Bed Slope S from the following relation

f 5/3
S=
3340Q 1/6
Example

Design an irrigation canal in alluvial soil according to Lacey’s silt theory for the following data.
Full supply discharge = 10 cumec
Lacey’s silt factor = 0.9
Side slopes of the channel = 21 (H):1(V)
Solution h 2 i1/6 h i1/6
Qf 10×0.92
Velocity V = 140 = 140 = 0.62 m/s
Area A = VQ = 0.62
10
= 16.13m2
2 (0.62)2
Hydraulic mean radius R = 25 Vf = 5
2 × 0.9 = 1.07 m
Perimeter √P = RA = 16.13
1.07 = 15.07m
P = B +√D 5
orB + D 5 = 15.07

A = BD + D 2 /2
or
BD + D 2/2 = 16.13
From the two equations, we get

B = 12.27m and D = 1.25m


√ √
Check: P √
= 4.75 Q=4.75 √ 10 = 15.02m
P = B + D 5 = 12.27 + 1.25 5 = 15.07m
Hence checked.

f 5/3 (0.9)5/3 1
Bed SlopeS = 3340(Q)1/6
= 3340(10)1/6
= 5844
Canal Irrigation

March 16, 2021

Canal Irrigation March 16, 2021 1 / 31


Table of Contents
1 PLANNING OF AN IRRIGATION CANAL SYSTEM
Preminary survey
Detailed survey
2 Alingment of irrigation Canals
3 Alingment of irrigation Canals
4 CANAL LOSSES
5 CURVES IN CANALS
6 ESTIMATION OF DESIGN DISCHARGE OF A CANAL
7 CANAL OUTLETS
Types of Canal Outlets
Parameters for Studying the Behaviour of Outlets
Flexibility
Sensitivity
Non-Modular outlets
Semi-Modular Outlets
Kennedys Gauge Outlet
Open Flume Outlet
orifice semi-module
Modular Outlets
Gibbs rigid module
Khannas rigid module

Canal Irrigation March 16, 2021 2 / 31


Preminary survey

Planning of an irrigation canal project includes the determination of: (i) canal alignment,
and (ii) the water demand. The first step in the planning of an irrigation canal project is to
carry out a preliminary survey to establish the feasibility of a proposal. Once the feasibility
of the proposal has been established, a detailed survey of the area is carried out and,the
alignment of the canal is fixed. The water demand of the canal is, then, worked out.
The information on the following features of the area are to be collected: (i) Type of soil,
(ii) Topography of the area, (iii) Crops of the area, (iv) Rainfall in the area, (v) Water
table elevations in the area, (vi) Existing irrigation facilities, and (vii) General outlook of
the cultivators with respect to cultivation and irrigation

Canal Irrigation March 16, 2021 3 / 31


Detailed survey

The preparation of plans for a large canal project is simplified in a developed area because
of the availability of settlement maps (also called shajra maps having scale of 16 inches
to a mile i.e., 1/3960 1/4000) and revenue records in respect of each of the villages of
the area. The settlement maps show the boundaries and assigned numbers of all the fields
of the area, location of residential areas, culturable and barren land, wells, ponds, and
other features of the area. Usually for every village there is one settlement (or shajra)
map. These maps and the revenue records together give information on total land area,
cultivated area, crop-wise cultivated area and the area irrigated by the existing ponds and
wells. With the help of settlement maps of all the villages in a doab, a drawing indicating
distinguishing features, such as courses of well-defined drainages of the area, is prepared.
On this drawing are then marked the contours and other topographical details not available
on the settlement maps but required for the planning of a canal irrigation project. Contours
are marked after carrying out levelling survey of the area. The details obtained from the
settlement maps should also be updated in respect of developments such as new roads,
additional cultivated area due to dried-up ponds, and so on.

Canal Irrigation March 16, 2021 4 / 31


Alingment of irrigation Canals
On projects where land slopes are relatively flat and uniform, it is advantageous to align
channels on the watershed of the areas to be irrigated. The natural limits of command of
such irrigation channels would be the drainages on either side of the channel. Aligning a
canal (main, branch as well as distributary) on the watershed ensures gravity irrigation on
both sides of the canal. Besides, the drainage flows away from the watershed and, hence,
no drainage can cross a canal aligned on the watershed.

Figure: Head reach of a main canal in plains


Canal Irrigation March 16, 2021 5 / 31
Alingment of irrigation Canals
In hilly areas, the conditions are vastly different compared to those of plains. Rivers flow
in valleys well below the watershed or ridge, and it may not be economically feasible to
take the channel on the watershed. In such situations, contour channels are constructed.
Contour channels follow a contour while maintaining the required longitudinal slope. It
continues like this and as river slopes are much steeper than the required canal bed slope
the canal encompasses more and more area between itself and the river.

Canal Irrigation March 16, 2021 6 / 31


CANAL LOSSES

Seepage water lost from the canals percolate deep into the ground and is the main cause
of the loss of water carried by a canal. In addition, some canal water is also lost due to
evaporation. The loss due to evaporation is about 10 per cent of the quantity lost due to
seepage. The seepage loss varies with the type of the material through which the canal
runs. Obviously, the loss is greater in coarse sand and gravel, less in loam, and still less
in clay soil. Between the headworks of a canal and the watercourses, the loss of water on
account of seepage and evaporation is also known as conveyance loss, and expressed as
m3 /s per million square metres of either wetted perimeter or the exposed water surface
area. This loss may vary from 20 to 50 per cent of water diverted at the headworks
depending upon the type of soil through which canal runs and the climatic conditions of
the region. The following empirical relation has been found to give comparable results
2
ql = (1/1200)(B + h) 3 (1)
In this relation, ql is the loss expressed in m3/s per kilometre length of canal and B and h
are, respectively, canal bed width and depth of flow in metres

Canal Irrigation March 16, 2021 7 / 31


Conveyance loss can be calculated using the values given in Table below.

Figure: Conveyance losses in canals

Canal Irrigation March 16, 2021 8 / 31


CURVES IN CANALS

The canal alignment does not remain straight all through the length of the canal, and
curves or bends have to be provided. There is a slight drop in the water surface at the
inner edge of the curve and a slight rise at the outer edge of the curve. This results
in slight increase in the velocity at the inner edge and slight decrease in the velocity at
the outer edge. As a result of this, the low-velocity fluid particles near the bed move to
the inner bank and the high-velocity fluid particles near the surface gradually cross to the
outer bank. The cross currents tend to cause erosion along the outer bank. Therefore,
wherever possible, curves in channels excavated through loose soil should be avoided. If it
is unavoidable, the curves should have a long radius of curvature.

Figure: Radius of curvature for channel curves

Canal Irrigation March 16, 2021 9 / 31


ESTIMATION OF DESIGN DISCHARGE OF A CANAL

The amount of water needed for the growth of a crop during its entire crop-growing period
is known as the water requirement of the crop, and is measured in terms of depth of water
spread over the irrigated area. This requirement varies at different stages of the growth of
the plant. The peak requirement must be obtained for the period of the keenest demand.
One of the methods to decide the water requirement is on the basis of kor watering.
When the plant is only a few centimetres high, it must be given its first watering, called
the kor watering, in a limited period of time which is known as the kor period. If the plants
do not receive water during the kor period, their growth is retarded and the crop yield
reduces considerably. The kor watering depth and the kor period vary depending upon the
crop and the climatic factors of the region. In UP, the kor watering depth for wheat is
13.5 cm and the kor period varies from 8 weeks in north-east UP (a relatively dry region)
to 3 weeks in the hilly region (which is relatively humid). For rice, the kor watering depth
is 19 cm and the kor period varies from 2 to 3 weeks.

∆ = 8.64b/D(meters) (2)
For the purpose of designing on the basis of the keenest demand (i.e., the kor period
requirement) the base period b and the water depth ∆ are replaced by the kor period and
kor water depth, respectively.

Canal Irrigation March 16, 2021 10 / 31


Example: The culturable command area for a distributary channel is 10,000 hectares. The
intensity of irrigation is 30 per cent for wheat and 15 per cent for rice. The kor period for
wheat is 4 weeks, and for rice 3 weeks. Kor watering depths for wheat and rice are 135
mm and 190 mm, respectively. Estimate the outlet discharge.

Since the water demands for wheat and rice are at different times, these are not cumulative.
Therefore, the distributary channel should be designed for the larger of the two discharges,
viz., 1.7 m3/s. The above calculations exclude channel losses and the water requirement
of other major crops during their kor period.

Canal Irrigation March 16, 2021 11 / 31


CANAL Outlets

When the canal water has reached near the fields to be irrigated, it has to be transferred
to the watercourses. An outlet is a masonry structure through which water is admitted
from the distributary into a watercourse. It also acts as a discharge measuring device. The
discharge though an outlet is usually less than 0.085 m3 /s.If the total available supply is
insufficient, the outlets must be such that equitable distribution can be ensured.
Types of Outlet Canal outlets are of the following three types: (i) Non-modular outlets,
(ii) Semi-modular outlets, and (iii) Modular outlets.

Non-modular outlets The non-modular outlets are very suitable for low head conditions
as the loss of head in this outlet is less than that in a modular outlet. The discharge in
non-modular outlets capacity depends on the difference of water levels in the distributary
and the watercourse. The discharge through these outlets fluctuates over a wide range
with variations in the water levels of either the distributary or the watercourse. Such an
outlet is controlled by a shutter at its upstream end.

Canal Irrigation March 16, 2021 12 / 31


CANAL OUTLETS

Semi-modular outlets The discharge through a semi-modular outlet (or semi-module or


flexible outlet) depends only on the water level in the distributary and is unaffected by
the water level in the watercourse provided that a minimum working head required for its
working is available. A semi-module is more suitable for achieving equitable distribution
of water at all outlets of a distributary. The only disadvantage of a semi-modular outlet is
that it involves comparatively greater loss of head.

Modular outlets Modular outlets are those whose discharge is independent of the water
levels in the distributary and watercourse. A modular outlet supplies fixed discharge and,
therefore, enables the farmer to plan his irrigation accordingly. However, in case of excess
or deficient supplies in the distributary, the tail-end reach of the distributary may either
get flooded or be deprived of water. This is due to the reason that the modular outlet
would not adjust its discharge corresponding to the water level in the distributary. But, if
an outlet is to be provided in a branch canal which is likely to run with large fluctuations
in discharge, a modular outlet would be an ideal choice.

Canal Irrigation March 16, 2021 13 / 31


Parameters for Studying the Behaviour of Outlets
Flexibility
The ratio of the rate of change of discharge of an outlet (dQ0 /Q0 ) to the rate of change
of discharge of the distributary channel (dQ/Q) (on account of change in water level) is
termed the flexibility which is designated as F. Thus
dQ0 /Q0
F = (3)
dQ/Q
Here, Q and Q0 are the flow rates in the distributary channel and the watercourse,respectively.
Discharge Q in the distributary channel is expressed as

Q = C1 hn (4)

dQ dh
=n (5)
Q h
Similarly, the discharge Q0 through the outlet can be expressed as
Q0 = C1 H m (6)

dQ0 dH
=m (7)
Q0 H
Here, m and n are suitable indices and C1 and C2 are constants.
Canal Irrigation March 16, 2021 14 / 31
Flexibility
Thus,

m h dH
F = x x (8)
n H dh
For semi-modular outlets, the change in the head dH at an outlet would be equal to the
change in the depth of flow dh in the distributary. Therefore,
m h
F = x (9)
n H
If the value of F is unity, the rate of change of outlet discharge equals that of the distributary
discharge. For a modular outlet, the flexibility is equal to zero. Depending upon the value
of F, the outlets can be classified as: (i) proportional outlets (F = 1), (ii) hyperproportional
outlets (F > 1), and (iii) subproportional outlets (F < 1). When a certain change in the
distributary discharge causes a proportionate change in the outlet discharge, the outlet (or
semi-module) is said to be proportional. A proportional semi-module ensures proportionate
distribution of water when the distributary discharge cannot be kept constant. For a
proportional semi-modular outlet (F = 1),

m h
= (10)
n H
The ratio (H/h) is a measure of the location of the outlet and is termed setting.
Canal Irrigation March 16, 2021 15 / 31
Flexibility

The setting for a proportional outlet is equal to the ratio of the outlet and the channel
indices.
For hyper-proportional and sub-proportional outlets the setting must be, respec-
tively,less and more thanm/n.
For a wide trapezoidal (or rectangular) channel,n can be approximately taken as 5/3
and for an orifice type outlet, m can be taken as 1/2. Thus, an orifice-type module
will be proportional if the setting (H/h) is equal to (1/2)/(5/3), i.e., 0.3.
The module will be hyper-proportional if the setting is less than 0.3 and sub-proportional
if the setting is greater than 0.3.
Similarly, a free flow weir type outlet (m = 3/2) would be proportional when the
setting equals 0.9 which means that the outlet is fixed at 0.9 h below the water
surface in the distributary.

Canal Irrigation March 16, 2021 16 / 31


Parameters for Studying the Behaviour of Outlets

Sensitivity The ratio of the rate of change of discharge (dQ0 /Q0 ) of an outlet to the rate
of change in the water surface level of the distributary channel with respect to the depth
of flow in the channel is called the sensitivity of the outlet. Thus,
dQ0 /Q0
S= (11)
dG /h
dQ0 /Q0
F = (12)
dQ/Q
dh
= (dQ0 /Q0 )/n( ) (13)
h
1
= S (14)
n
S = nF (15)

Canal Irrigation March 16, 2021 17 / 31


Non-modular outlet

The non-modular outlet is usually in the form of a submerged pipe outlet or a masonry
sluice‘which is fixed in the canal bank at right angles to the direction of flow in the
distributary.
The diameter of the pipe varies from 10 to 30 cm. The pipe is laid on a light concrete
foundation to avoid uneven settlement of the pipe and consequent leakage problems.
The pipe inlet is generally kept about 25 cm below the water level in the distributary.
When considerable fluctuation in the distributary water level is anticipated, the inlet
is so fixed that it is below the minimum water level in the distributary.

Canal Irrigation March 16, 2021 18 / 31


Non-modular outlet

Pipe outlet

Figure: Pipe outlet

Canal Irrigation March 16, 2021 19 / 31


Non-modular outlet

If H is the difference in water levels of the distributary and the watercourse then the
discharge Q through the outlet can be obtained from the equation,

V2 fL
H= [0.5 + + 1] (16)
2g D
where
Q p d
V = 2
= 2gH( )1/2 (17)
(π/4)d 1.5d + fL
where, d = diameter of pipe outlet L = length of pipe outlet and f = friction factor for
pipe. Alternatively, the discharge Q can be expressed as

Q = AV (18)
π 2p d
= d 2gH( )1/2 (19)
4 1.5d + fL
p
Q = CA 2gH (20)
d
C =( )1/2 (21)
1.5d + fL

Canal Irrigation March 16, 2021 20 / 31


Semi-Modular Outlets (Semi-Modules or Flexible Outlets)

The simplest type of semi-modular outlet is a pipe outlet discharging freely into the
atmosphere.
The pipe outlet, described as the non-modular outlet, works as semi-module when it
discharges freely into the watercourse.
The exit end of the pipe is placed higher than the water level in the watercourse.
In this case, the working head H is the difference between the water level in the
distributary and the centre of the pipe outlet.
The efficiency of the pipe outlet is high and its sediment conduction is also good.
The discharge through the pipe outlet cannot be increased by the cultivator by digging
the watercourse and thus lowering the water level of the watercourse.
Usually, a pipe outlet is set so that it behaves as subproportional outlet, i.e., its setting
is kept less than 0.3.

Canal Irrigation March 16, 2021 21 / 31


Kennedys Gauge Outlet

It mainly consists of an orifice with bellmouth entry, a long expanding delivery pipe,
and an intervening vertical air column above the throat
The air vent pipe permits free circulation of air around the jet. This arrangement
makes the discharge through the outlet independent of the water level in the water-
course.
The water jet enters the cast iron expanding pipe which is about 3 m long and at the
end of which a cement concrete pipe extension is generally provided. Water is then
discharged into the watercourse.
This outlet can be easily tampered with by the cultivator who blocks the air vent pipe
to increase the discharge through the outlet.

Canal Irrigation March 16, 2021 22 / 31


Kennedys Gauge Outlet

Figure: Kennedys Gauge Outlet

Canal Irrigation March 16, 2021 23 / 31


Open Flume Outlet

An open flume outlet is a weir with a sufficiently constricted throat to ensure super-
critical flow and long enough to ensure that the controlling section remains within the
throat at all discharges up to the maximum.
A gradual expansion is provided downstream of the throat. The entire structure is
built in brick masonry but the controlling section is generally provided with cast iron
or steel bed and check plates.
This arrangement ensures the formation of hydraulic jump and, hence, the outlet
discharge remains independent of the water level in the watercourse.
The discharge through the outlet is proportional to H3/2. The efficiency of the outlet
varies between 80 and 90 per cent.
For the range of outlet discharges normally used, the outlet is either deep and narrow,
or shallow and wide. While a narrow outlet gets easily blocked, a shallow outlet is not
able to draw its fair share of sediment.

Canal Irrigation March 16, 2021 24 / 31


Open Flume Outlet

Figure: Open Flume Outlets

Canal Irrigation March 16, 2021 25 / 31


Orifice semi-module

An orifice semi-module consists of an orifice followed by a gradually expanding flume


on the downstream side .
Supercritical flow through the orifice causes the formation of hydraulic jump in the
expanding flume and, hence, the outlet discharge remains independent of the water
level in the watercourse.
The roof block is suitably shaped to ensure converging streamlines so that the dis-
charge coefficient does not vary much. The roof block is fixed in its place by means
of two bolts embedded in a masonry key.
Tampering with the outlet by the cultivators would be easily noticed through the
damage to the masonry key. This is the chief merit of this outlet.

Canal Irrigation March 16, 2021 26 / 31


Orifice semi-module

Canal Irrigation March 16, 2021 27 / 31


Modular Outlets

Most of the modular outlets have moving parts which make them costly to install as well
as maintain. The following two types of modular outlets (also known as rigid modules),
however, do not have any moving part:
1 Gibbs rigid module, and
2 Khannas rigid module.

Gibbs rigid module This module has an inlet pipe under the distributary bank. This
pipe takes water from distributary to a rising spiral pipe which joins the eddy chamber
. This arrangement results in free vortex motion. Due to this free vortex motion,
there is heading up of water near the outer wall of the rising pipe. The water surface
thus slopes towards the inner wall. A number of baffle plates of suitable size are
suspended from the roof of the eddy chamber such that the lower ends of these plates
slope against the flow direction. With the increase in head, the water banks up at
the outer wall of the eddy chamber and impinges against the baffles and spins round
in the compartment between two successive baffle plates. This causes dissipation of
excess energy and results in constant discharge. The outlet is relatively more costly
and its sediment withdrawal is also not good.

Canal Irrigation March 16, 2021 28 / 31


Figure: Gibbs rigid module

Canal Irrigation March 16, 2021 29 / 31


Modular Outlets
Khannas rigid module
This outlet is similar to an orifice semi-module. But, in addition, it has sloping shoots
fixed in the roof block . These shoots cause back flow and thus keep the outlet discharge
constant. If the water level in the distributary is at or below its normal level, the outlet
behaves like an orifice semi-module. But, when the water level in the distributary channel
is above its normal level, the water level rises in chamber A, and enters the first sloping
shoot. This causes back flow and dissipates additional energy. This maintains a constant
discharge. The number of sloping shoots and their height above the normal level can vary
to suit local requirements. The shoots are housed in a chamber to prevent them from
being tampered with. If the shoots are blocked, the outlet continues to function as a
semi-module.

Figure: Khannas rigid module


Canal Irrigation March 16, 2021 30 / 31
Modular Outlets

Canal Irrigation March 16, 2021 31 / 31


Diversion Headworks
Introduction

An irrigation canal takes its supplies from a river or a stream. In order to divert water from the
river into the canal, it is necessary to construct certain works or structures across the river and at
the head of the offtaking canal. These are known as canal headworks. THe canal headworks may
be classified into the following two types.
Storage headworks
Consists of a dam constructed across the river to create a reservoir in which water is stored
during the period of excess flow in the river.
Diversion headworks
It serves to raise the water level in the river and divert the required quantity into the canal.
Command area is increased.
Regulates the supply of water into the canal.
Controls the entry of silt into the canal.
Provides some storage of water for a short period.
Reduces the fluctuations in the level of supply in the river.
Types of diversion headworks

Temporary diversion headworks


Consists of a spur or bund constructed across a river to raise the water level in the river and
divert it into the canal.
Permanent diversion headworks
Consists of a permanent structure such as weir or barrage cosntructed across the river to
raise the water level in the river and divert it into the canal. Most of the diversion
headworks for important canal system are permanent diversion headworks.
Components

The various components of a diversion headworks are as follows:


Weir or barrage
Divide wall
Fish Ladder
Pocket or approach channel
Undersluices or scouring sluices
Silt excluder
Canal Head regulator
River training works such as Marginal bunds or Guide bunds
Marginal Bund Guide Bund

River

Divide Wall

Head Regulator
Approach Channel
Canal

Weir
Scouring Sluices

Fish Ladder

Typical Layout of Diversion Headworks


Figure:
Undersluices

The undersluices are openings provided in the weir wall with their crest at a low level. They are
located on the same as offtaking canal.
Functions of undersluices
They preserve a clear and well-defined channel towards the head regulator.
They scour the silt deposited on the river bed in the pocket upstream of the canal head
regulator.
They pass low floods without the necessity of dropping the weir crest shutters
Divide Wall

A divide wall is a long masonry wall which is constructed at right angles to the axis of the weir to
seperate the undersluices from the rest of the weir. The top width of divide wall is about 1.5 to
2.5m.
Fish ladder

Large rivers have many types of fishes. They move from one part to another according to season.
In our country generally fish move from upstream to downstream in search of warmth and return
upstream before monsoon for clear water. Due to the construction of a weir across the river, such
migration will be obstructed and large scale destruction of fish life may take place in the river.
Thus to enable the fish to migrate, fish ladder is provided. Since most fish can travel upstream
only if the velocity of flow does not exceed 3 to 3.5 m/s, the design of the fish ladder should be
such that it constantly releases water at a velocity not exceeding this value.
Canal Head regulator

A canal head regulator is a structure constructed at the head of the canal taking
off from the upstream of a weir or barrage. It consists of a number of spans
supported by piers which support the gates provided for regulation of flow into
the canal.
Functions
It regulates the supply of water into the canal.
It controls te entry of silt into the canal.
It excludes the high flood from entering the canal.
Weir

A weir is a structure constructed across a river to raise its water level and divert the water into the
canal. On the crest of the weirs usually shutters are provided so that part of the raising up of
water (or ponding) is carried out by shutters. During floods, the shutters may be dropped down to
allow water to flow over the crest of the weir. Weirs are usually aligned at right angles to the
direction of flow of the river.
Weirs may be classified according to the material of construction and certain design features into
the following three types
Masonry weirs with vertical drop weir
Rockfill weirs
Concrete weirs
Pond Level
Crest Shutter

U/S Pile Intermediate Pile


D/S Pile

Figure: Concrete Weir


Crest Shutter
Pond Level

Weir Wall
Block Protection
Inverted Filter

U/S PILE Launching Apron


D/S PILE

Vertical Drop Weir


Figure:
Vertical drop weir

Impervious horizontal floor or apron and a masonry weir wall with with either both faces
inclined or both faces vertical or one face vertical and other face inclined
At the u/s and d/s ends, piles are provided
At the u/s end,a block protection and at the d/s end, a graded filter is provided
Launching aprons or pervious aprons are provided after the block protection or filter
Suitable for any type of foundation
Floor design based on Bligh’s theory
Barrage

The crest is kept at a low level and the raising up of water level is accomplished mainly by means
of gates. During floods, these gates can be raised clear off the high flood level and thus enable the
high flood to pass with mimimum of afflux. It provides a better control on the water level in the
river but is comparatively more costly.Design involves same procedure as a concrte weir.
High Flood Level

Pond Level
GATE

Inverted Filter

Launching Apron

U/S PILE

D/S PILE
Figure:
Causes of failure if Weirs on permeable foundation and their remedies

The various causes of failure of weirs on permeable foundations may be classified broadly into two
categories:
Due to seepage flow
Due to surface flow
The seepage flow may cause the failure of a weir in two ways
By piping
If the water percolating through the foundation has sufficient force when it emerges at the
downstream end of the impervious floor it may lift up the soil particles at the end of the floor
With the removal of surface soil, there is further concentration of flow into the resulting
depression and more soil is removed.
This process of erosion progressively extends backwards towards the u/s side and results in
the removal of soil and developing pipe like formation beneath the floor.
The floor may subside in the hollows so formed and fail which is known as failure due to
piping.
The following measures may be taken to prevent the failure due to piping
Providing sufficient length of the impervious floor so that the path of percolation is increased
and exit gradient(hydraulic gradient) is reduced.
Providing piles at the upstream and downstream ends of the impervious floor.
Causes of failure if Weirs on permeable foundation and their remedies

By Uplift Pressure
The water percolating through the foundation exerts an upward pressure on the imprevious
floor. This pressure is known as uplift pressure. If the uplift pressure is not counterbalanced by
the weight of the floor, it may fail by rupture.
The following measure may be taken to prevent the failure of the floor by rupture due to excessive
uplift pressure.
Providing sufficient thickness of the impervious floor.
Providing pile at the u/s end of the impervious floor so that the uplift pressure is reduced in
the d/s side.
Barrier
(H/L)2d1
(H/L)2d2
Subsoil hydraulic
gradient Line H

(H/L)2d3

d2
d1

d3

Bligh’s Path of Creep


Figure:
Bligh’s Creep Theory

Bligh assumed that the percolating water creeps along the base profile of the structure which is in
contact with the subsoil. The length of path thus traversed by the percolating water is called the
length of creep. He further assumed that the heas loss per unit length of creep which is called
hydraulic gradient is constant throughout the percolating passage which means loss of head is
proportional to the length of creep. He made no distinction between vertical and horizontal creep.
In the figure, a barrier impounding water of depth H is provided with a horizontal floor of length b
with three vertical cutoffs (sheet piles). The percolating water then follows the path indicated by
arrows and the creep length L will be given by

L = b + 2d1 + 2d2 + 2d3


The hydraulic gradient or the loss of head per unit length of creep is given by the following
expression
H H
= (1)
L b + 2d1 + 2d2 + 2d3
As hydraulic gradient is constant, if L1is the creep length upto any point, then head loss upto this
point will be (H/L)L1and the residual head at this point will be [H − (H/L)L1]. In the planes of the
vertical cutoffs, there will be losses of head equal to (H/L)2d1, (H/L)2d2and (H/L)2d3.
The reciprocal of hydraulic gradient i,e,(L/H) is known as Bligh’s coefficient of creep C. Thus
L = CH (2)
Bligh’s Creep Theory

To ensure the safety of the impervious floor against the two possible ways in which failure may be
caused by subsurface flow, following criteria are required to be satisfied.
Safety against piping
The length of creep should be sufficient to provide a safe hydraulic gradient according to the
type of soil. He has recommended certain values of C for different soils as given in Table.
According to Bligh, if the hydraulic gradient ≤ (1/C ), there will be no danger of piping.
The seepage head H is to be measured from the water level upstream to the corresponding
lowest water level downstream.
Type of soil Value of C Safe hydraulic gradient
Light sand and mud 18 1/18
Fine mica sand 15 1/15
Coarse grained sand 12 1/12
Sand mixed with boulder and gravel 5 to 9 1/9 to 1/5
Bligh’s Creep Theory

Safety against uplift pressure


The ordinate of the subsoil hydraulic gradient line above the bottom of the floor at any point represents the uplift
pressure at that point.
If at any point, h’ is the ordinate of the hydraulic gradient line above the bottom of the floor, then at this point, the
uplift pressure exerted by the percolating water is wh’, where w is the specific weight of water.
If floor thickness is t and the specific gravity of the material of the floor is G, the downward force per unit area due
to the weight of the floor is (wG)t.
For equilibrium, the uplift pressure must be counterbalanced by the weight of the floor. Thus equating two,’

wh0 = (wG )t

h0
t= (3)
G

h0 = tG

∴ h0 − t = tG − t = t(G − 1)

h0 − t
∴t=
G −1
h
or t = (4)
G −1
The floor thickness given by the above equation is usually incrased by considering a factor of safety of (4/3) and hence
4 h
t= (5)
3 G −1
Example

Subsoil hydraulic
5m gradient Line

A B C
5m 3m
5m
8m 10 m
10 m 8m

25 m

Figure:

The figure shows the section of a hydraulic structure on permeable foundation. Calculate the
average hydraulic gradient according to Bligh’s creep theory. Also find the uplift pressures at
points A,B and C as shown in the figure and the floor thickness required at these points.
Example

Solution
According to Bligh’s creep theory, the total creep length is given by

L = (2 × 5) + (2 × 3) + (2 × 10) + 25 = 61 m

5 1
∴Hydraulic gradient = 61 = 12.2
The structure would be safe on coarse grained sand.
Uplift pressure at point A
Length of creep upto point A = (2×5)+5 = 15
Residual seepage head at point A
15
h1 = 5(1 − ) = 3.77 m
61

∴Uplift pressure at point A = wh1= 9.81 × 3.77 = 36.984kN/m2


The required thickness of the floor at point A is

4 h 4 3.77
t1 = = = 4.05m
3 G − 1 3 2.24 − 1

Uplift pressure at point B


Length of creep upto point B = (2×5)+(2×3)+10 = 26 m
Residual seepage head at point B
26
h1 = 5(1 − ) = 2.87 m
61

∴Uplift pressure at point B = wh1= 9.81 × 2.87 = 28.155kN/m2


The required thickness of the floor at point B is

4 h 4 2.87
t1 = = = 3.09m
3 G − 1 3 2.24 − 1

Uplift pressure at point C


Length of creep upto point C = (2×5)+(2×3)+18= 34m
Residual seepage head at point C
34
h3 = 5(1 − ) = 2.21 m
61

∴Uplift pressure at point C = wh1= 9.81 × 2.21 = 21.68 kN/m2


The required thickness of the floor at point C is

4 h 4 2.21
t1 = = = 2.38m
3 G − 1 3 2.24 − 1
Khosla’s Theory

A number of important structures were designed on the basis of this theory, someof which
remained stable while others gave trouble or failed. Khosla carried out investigation and led to the
following conclusions.
The outer faces of the end sheet piles were much effective than the inner ones and horizontal
length of the floor.
The intermediate sheet piles, if smaller in length than the outer ones were ineffective except for
local redistrbution of pressures.
Undermining of floors started from the tail end. If the hydraulic gradient at the exit was more
than the critical gradient for the particular soil, the soil particles would move with the flow of
water thus causing progressive degradation of the subsoil, resulting in cavities and ultimate
failure.
It was absolutely essential to have a reasonably deep vertical cutoff at the d/s end of the floor
to prevent undermining.
Khosla’s method of independent Variables

W.S. W.S.

H H

E1 C1 E C
d
d
b b
D1
D
Pile at Upstream end Pile at Downstream end

W.S.

E C
b1
d

Intermediate Pile
Figure:
The above cases have been analysed by Khosla for determining the residual seepage head or the uplift pressure head at
the key points and the exit gradient. The key points are the junction points of pile and floor, the bottom point of pile
and the bottom corners of depressed floor. The results of this analysis are presented in the form of curves from which the
values of φi.e., the ratio of the residual seepage head and the total seepage head can be determined at the key points.
Pile at Upstream end

100 λ−2
φC1 = 100 − cos−1
π λ

100 λ−1
φD1 = 100 − cos−1
π λ

where

1+ 1 + α2
λ=
2

b
α=
d

Pile at Downstream end

100 λ−2
φE = cos−1
π λ

100 λ−1
φD = cos−1
π λ
Intermediate Pile

100 λ1 − 1
φE = cos−1
π λ

100 λ1
φD = cos−1
π λ

100 λ1 + 1
φC = cos−1
π λ

b
α1 =
d

b − b1
α2 =
d

q q 
λ1 = 1 + α12 − 1 + α22 /2

q q 
λ1 = 1 + α12 + 1 + α22 /2
Corrections

Figure:

Ec E c t2 t3 E c
t1

c1 E1 c1 E1
Flow Flow
d3
d1 d2
Flow

D D D

Intermediate Pile Pile At Downstream end


Pile At Upstream end

Correction for floor thickness


Correction for thickness of floor

In the standard forms with vertical cutoffs, the thickness of the floor is assumed to be negligible. So the pressures at
the junction points E and C pertain to the level at the top of the floor whereas the actual junction is with the bottom
of the floor. The pressures at the actual junction points E1and C1are interpolated by assuming a linear variation from
the hypothetical point E to D and also from D to C.

For pile at the upstream end


Correction for C1= φDd−φ
1
C
×t1 (additive)
∴Pressure at C1

φD − φC
φC1 = φC + × t1
d1

where t1 = Floor thickness


d1 = depth of pile

For intermediate pile


Correction for E1= φE d−φ
2
D
× t2 (subtractive)
∴Pressure at E1

φE − φD
φE1 = φE − × t2
d2

Correction for C1= φDd−φ


2
C
×t2 (additive)
∴Pressure at C1

φD − φC
φC1 = φC + × t2
d2
For pile at downstream end
Correction for E1= φE d−φ
3
D
× t3 (subtractive)
∴Pressure at E1

φE − φD
φE1 = φE − × t3
d3
Correction for mutual interferecence of piles

Correction is given by following expression


r  
D d +D
C = 19
b0 b

where C = correction to be applied as percentage of head


b’ = distance between piles
b = total length of floor
D = depth of pile whose effect is required to be determined on neighbouring pile of depth d
d = depth of pile on which the effect of pile of depth D is required to be determined.
The correction is additive for points in the backwater and subtractive for points forward in the
direction of flow. So the effect of interference of pile no. 2 on pile no. 1 is determined, the
correction will be additive since point C of pile no. 1 is in the rear with respect to the direction of
flow. Pile no. 2 causes a blocking effect on the subsoil flow which results in increase in pressure at
all points on the upstream side. The effect of interference of pile is to be determined only for that
face of the adjacent pile which is towards the interfering pile, e.g., pile no. 2 will interfere with the
d/s face of pile no.1 and u/s face of pile no. 3.
Pond Level
Crest Shutter

EC

Pile 1 d EC
EC
bs
Pile 2
D D
b’
Pile 3
D
D
b

Figure: Correction for mutual interference of piles


Correction for slope of the floor

A suitable correction is to be applied for a sloping floor. The values of the correction in percent of
pressure are given in the following table.
Slope (Vertical to Horizontal) Correction % of pressure
1 in 1 11.2
1 in 2 6.5
1 in 3 4.5
1 in 4 3.3
1 in 5 2.8
1 in 6 2.5
1 in 7 2.3
1 in 8 2.0

The correction given by this table is to be multiplied by the horizontal length of the sloping floor
and divided by th distance between the two pile lines between which the sloping floor is located.
The correction is applicable only to those key points of the pile lines at the beginning or the end
of the sloping floor which are facing the sloping floor. So in the above figure, the correction for
slope is to be applied only to point E of pile no. 2 and the correction will be obtained multiplying
the appropriate value from the table by (bs /b’). The correction is additive for a downward sloping
floor and subtractive for an upward sloping floor in the direction of flow.
The uplift pressure distribution along the floor may be obtained by assuming a linear variation
between the key points after obtaining pressures at the key points.
Exit Gradient

Defined as the hydraulic or pressure gradient of subsoil flow at the downstream or the exit end
of the floor.
For a length of floor of length b with a vertical cutoff of depth d at its downstream end, Khosla
and his associates derived an expression for the exit gradient GE as follows:
H 1
GE = √
dπ λ
where H = total seepage head

1 + 1 + α2
λ=
2
and α = db
From the above equation, it can be seen that if no cutoff is provided at the downstream end of
the floor, i.e., d = 0, the exit gradient GE is infinite. But it is evident that if there is no cutoff at
the d/s end, a higher exit gradient will exist will exist which may lead to the failure of the floor
due to piping. It is essential to provide a vertical cutoff at the d/s end of the floor to reduce the
exit gradient. The exit gradient should always be less than critical hydraulic gradient which is
defined as the gradient at which the soil particles will be lifted up and lead to undermining. The
permissible exit gradients for different soils are given in the following table.

Type of soil Exit gradient


Fine soil 1/6 to 1/7
Coarse sand 1/5 to 1/6
Figure:
Example

Using Khosla’s method, obtain the uplift pressures for the weir profile shown in the figure. Also
calculate the exit gradient and find whether it is safe against piping if it is founded on fine sand
with permissible exit gradient 1/6.
Solution

Pile at Upstream end


100 λ−2
φC1 = 100 − cos−1 = 67.313%
π λ

100 λ−1
φD1 = 100 − cos−1 = 77.418%
π λ

where

1+ 1 + α2
λ= = 4.145
2

b 65
α= = = 7.222
d 100 − 91

Correction for floor thickness


φD1 − φC1 77.418 − 67.313
= × t1 = × 2 = 2.24%(additive)
d1 9

Correction for mutual interference of intermediate pile on u/s pile at C1


r  
D d +D
C = 19
b0 b

d = 98 − 91 = 7m
D = 98 − 89.5 = 8.5m

0
b = 16 + 32 = 48m
Solution(Contd.)

r  
8.5 7 + 8.5
C = 19 = 1.9%(additive)
48 65
∴Corrected value of φC1 = 67.313+2.24+1.9 = 71.453%
Pile at Downstream end
100 λ−2
φE3 = cos−1 = 27.976%
π λ

100 λ−1
φ D3 = cos−1 = 19.452%
π λ

where

1+ 1 + α2
λ= = 5.525
2

b 65
α= = = 10
d 6.5

Correction for floor thickness


φE3 − φD3 27.976 − 19.452
= × t3 = × 2 = 2.627%(subtractive)
d3 6.5
Solution(Contd.)

Correction for mutual interference of intermediate pile on d/s pile at E3


r  
D d +D
C = 19
b0 b

d = 94 − 89.5 = 4.5m

d = 94 − 89.5 = 4.5m

r  
4.5 4.5 + 4.5
C = 19 = 1.395%(subtractive)
16 65
∴Corrected value of φE3 = 27.976 − 2.627 − 1.395 = 23.954%
Solution(Contd.)

Intermediate pile
b1 48.5
α1 = = = 7.461
d 6.5

b − b1 65 − 48.5
α2 = = = 2.538
d 6.5

q q 
λ1 = 1+ α12 − 1+ α22 /2 = 2.401

q q 
2 2
λ= 1 + α1 + 1 + α2 /2 = 5.128

100 λ1 − 1
φE2 = cos−1 = 41.19%
π λ

100 λ1
φ D2 = cos−1 = 34.49%
π λ

100 λ1 + 1
φC2 = cos−1 = 26.91%
π λ
Solution(Contd.)

Correction for floor thickness


φE2 −φD2
Correction for E2 = d2
× t2 = 41.19−34.49
6.5
× 3 = 3.1%(−)
φD2 −φC2 34.49−26.91
Correction for C2 = d2 ×t2 = 6.5
3 = 3.5%(+)
×
Correction for interference of u/s pile at φE2
r   r  
D d +D 2 3.5 + 2
C = 19 = 19 = 0.3%(−)
b0 b 48 65

D = 93 − 91 = 2m

d = 93 − 89.5 = 3.5m
Correction for interference of d/s pile at φC2
r   r  
D d +D 3.5 3.5 + 3.5
C = 19 = 19 = 1%(+)
b0 b 16 65

d = 93 − 89.5 = 3.5m

D = 93 − 89.5 = 3.5m
Correction for slope
Slope = 97−93
32
=1 in 8
From the table, for a slope of 1 in 8, correction in % of pressure = 2%
32
∴Correction = 2× bbs0 =2× 48 = 1.3%(+)
∴Corrected value of φE2 = 41.19 − 3.1 − 0.3 + 1.3 = 39.1%
∴Corrected value of φC2 = 26.91 + 3.5 + 1 = 31.41%
Solution(Contd.)

Exit Gradient
H 1
GE = √
dπ λ
where H = total seepage head
H = 103 -96 = 7m
d = 96 - 89.5 = 6.5 m

b 65
α= = = 10
d 6.5


1+ 1 + α2
λ= = 5.525
2

7 1 1
∴ GE = × √ =
6.5 π 5.525 6.856

Permissible gradient = 16
Hence safe against piping.
Land Drainage

A properly designed drainage system is an effective means to prevent land from


getting waterlogged as well as to relieve the land already waterlogged. The
importance of drainage is recognised that it is constructed an integral part of the
irrigation scheme.
Before undertaking a drainage project, investigations should be carried out which
include topographical, geological and soil surveys. The nature of soils and
subsoils should also be studied. A knowledge of water table and its fluctutations
in the area proposed for irrigation is also essential.
Types of drains

The various types of drains used as anti-waterlogging measure is classified into


two categories:
I Open drains
I Shallow surface drains - These drains are used to drain away excess
irrigation water supplied to fields and also to quickly dispose storm water.
They therefore help to reduce percolation of water to groundwater reservoir
by reducing the detention of surface water and thus prevent waterlogging of
irrigated land. However they provide very little underdrainage.
I Deep open drains - These drains are use to drain out the subsoil and hence
useful for prevention of watelrogging of land as well as for relieving the land
already waterlogged. These drains are commonly used as outlet drains for a
closed drain system for the reclamation of waterlogged land.
I Closed drains
Design and maintenance of open drains

I Layout - Aligned along the path of the natural drainage and should be provided with an outfall,
either into a bigger drain or natural stream
I Capacity - Drains are provided to carry storm runoffs, seepage water from subsoil and surface
flow resulting from excess irrigation of fields. Since storm runoffs are usually far in excess of
the other flows, the drains are designed for storm runoff only. A judicious estimate of surface
runoff should be made to obtain the design capacity of drains.
I Section - The drains have trapezoidal cross-section and are designed on the same principle of
irrigation channels. Since deep drains receive both seepage and storm water, a small drain in
the bed of larger drain is usually provided to carry the small seepage discharge. With this
arrangement, the full section of the drain would be operativeonly during te storm. This would
reduce the problem of maintenance because the seepage water which would otherwise have
flown at small depth and velocity over the entire width leading to weed growth over the entire
section would flow only thorugh the small drain and so only this sction would be required to
maintained. The depth and bottom width of deep drains generally varies from 0.6 to 1.5 m and
0.3 to 1.2m respectively.
I Maintenance - The main problem of maintenance is to keep from weeds. It may be retarded by
having larger velocities of flow in the drains.
Drawbacks of open drains

I For the construction of open drains, valuable agricultural land is wasted.


I These drains obstruct the farming operations.
I Bridges are required to be constructed to facilitate communication across wide
drains.
I Valuable plant nutrients are washed down in the open drains.
Closed Drains

Closed drains are useful for prevention of waterlogging of land as well as for
relieving the land already waterlogged. These are tile drains usually of porous
earthen ware which are laid below the ground level butting each other with open
joints and covered up by earth. Thus these drains do not put any area out of
cultivation and do not obstruct any farming operations. The usual spacing of
these drains is from 15 to 45 m, closer spacing being used for soils of low
permeability. The usual diameter of tile drains is 100 mm. The drains are usually
located about 0.3m lower than the desired water table. These drains are laid at
at a gradient steeper than 1 in 500. As far as possible, the closed drains should
be placed in a permeable stratum to ensure effective drainge. The closed drains
have their outlets in natural or artificial channels.
Closed drains are designed to carry only the seepage water and their capacity is
determined by the rate of infiltration which can be estimated theoretically if the
soil permeabilities are known.
Layout of closed drains system

A closed drain system in general consists of a number of lateral drains which


collect water from different areas and discharge into main drain which is usually
an open drain. Sometimes laterals discharge into submain which is a closed drain
and the submain dischrages into main drain. The main drain in turn discharge
into outfall drain which takes water to natural drain. The layout is goverend by
the topography of the are to be drained. The various layouts for closed drain
system are given below.
I Natural system - In this system, the main/submain and the connecting laterals
are provided in the natural courses.
I Grid-iron layout - Laterals are provided only on one side of the main/submain.
The system is used where the land is practically level or where the land slopes
away from the main/submain on one side and when the entire area has to be
drained.
I Herringbone pattern - Laterals join main/submain from each side alternately.
It is adopted when the main/submain is laid into a depression. In this case,
the land along the main/submain is double drained which is however necessary
because it is in depression and requires more drainage than the land on
adjacent slopes.
Layout of closed drains system

I Double main system - Two seperate mains/submains with seperate laterals are
provided. The system is provided when the bottom of the depression is wide.
This arrangement helps to reduce the length of laterals.
I Intercepting drain system - In this system, there are no laterals but only a
main/submain is provided at the toe of the slope. The arrangement is adopted
when the hilly land is to be drained.
Layout of closed drains system
Discharge and spacing of closed drains

Figure:

With reference to the figure, let L be the spacing of the drains and a be their height above the
impervious stratum. Let b be the maximum height of the drained water table above the
impervious stratum. At a distance x from the centre of drain, let the drained water table be at a
height y above the impervious stratum. The various assumptions are as follows:
i) The hydraulic gradient at a distance x from the centre of the drain is (dy/dx).
ii) Flow lines are parallel and area of flow section at a distance x from the centre of the drain per
unit length of drain is y×1=y
iii) Discharge towards the drain is inversely proportional to the distance from the drain.
Discharge and spacing of closed drains

With the above assumptions according to Darcy’s law,

dy
Qy = ky (1)
dx
where Qy is the discharge passing throguh the section at a distance x from the centre of drain per
unit length of the drain.
If QD is the total discharge per unit length carried by the drain, then (QD /2)enters the drain from
either side. Since discharge is inversely proportional to the distance from the drain, discharge = 0
when x=(L/2) and discharge = (QD /2)when x=0.
Thus
QD (L/2) − x
Qy =
2 (L/2)

or
QD
Qy = (L − 2x) (2)
2L
Discharge and spacing of closed drains

Equating the value of Qy from equations 1 and 2, we get


QD dy
(L − 2x) = ky
2L dx

or
QD
(L − 2x)dx = ydy
2kL

Integrating we get,

QD y2
(Lx − x 2 ) = +C
2kL 2

When x=0, y=a



a2
C =−
2

Making this substitution, we get

Lk(y 2 − a2 )
QD =
Lx − x 2

When x=(L/2), y=b.

4k(b2 − a2 )
∴ QD = (3)
L

and
4k(b2 − a2 )
L= (4)
QD
Discharge and spacing of closed drains

From equations 3 and 4, discharge and spacing of closed drains can be determined. For a given
drainage system, equation 3 can be used to predict the inflow. However, to determine the spacing
for new drainage system, by equation 4 we can get if QD is known which will depend on infiltration
discharge into the ground which should be removed by drain. Different values have been
suggested for QD out of which the commonly adopted value being 1% of the average annual rainfall
drained in 24 hours.
Example 1

In a system of closed drains, the drains are placed with their centres 7.7 m above the impervious
stratum and the maximum height of the drained water table above the impervious stratum is 8m.
If the spacing between the drains is 25m and discharge entering the drain per unit length is
4.5×10−6 cumec/m, find the coefficient of permeability of soil.
Solution

4k(b2 − a2 )
L=
QD

or
LQD
k=
4(b2 − a2 )

L = 25m, QD = 4.5 × 10−6 cumec/m,


b = 8m, a = 7.7m
Substituing these values, we get,
k = 5.97 × 10−6 m/s
Example 2

In a drainage system, closed drains are to be placed with their centres 2 m below the ground level
to keep the highest position of the water table 1.7m below the ground level. The impervious
stratum is at a depth of 9.6m below the ground level. If the average annual rainfall in the area is
850 mm, find the spacing of drains. Assume 1% of the average annual rainfall to be drained in 24
hours and coefficient of permability k=1×10−5 m/s
Solution

850×A×1 3
1% of average annual rainfall over an area A km2 catered by each drain = 1000×100 m = 85 × 10−4 A m3
This is to be drained in 24 hours and hence the volume of water to be drained per second
−4
= 85×10 A
24×3600 cumec
If L is the spacing beyween the drains, then the area catered by drain per unit length = L m2

85 × 10−4 × L
∴ QD = = 9.84 × 10−8 L cumec/m
24 × 3600
From the equation (4), we have,
4k(b2 − a2 )
L=
QD
b=(9.6-1.7) = 7.9m and a = (9.6-2) = 7.6m
Subtituing these values,
4 × 1 × 10−5 [(7.9)2 − (7.6)2 ]
L=
9.84 × 10−8 L

L2 = 1890.24

∴ L = 43.48m = 43.5 m

Hence the drains hould be provided at a spacing of 43.5m.


Example 3

In a drainage system, closed drains are to be placed with their centres 10.5 m above the
impervious stratum and the maximum height of the drained water table above the centre of drains
is 0.3m. If the spacing of drains is 36m and the drrains carry 1% of the average annual rainfall in
24 hours, find the average annual rainfall. Take coefficient of permability k=5×10−6 m/s.
Solution

From the equation,

4k(b2 − a2 )
QD =
L
k= 5×10−6 m/s, b= (10.5+0.3) = 10.8m, a = 10.5 m and L= 36m

QD = 3.55 × 10−6 cumec/m

If the average annual rainfall is x mm, then the volume of water drained per second
x×A x×L
= 1000×100×24×3600 cumec = 1000×100×24×3600 cumec
Substituing the value of QD , we get

x × 36
3.55 × 10−6 =
1000 × 100 × 24 × 3600

∴ x = 852mm

The average annual rainfall is 852 mm.

You might also like